Sie sind auf Seite 1von 110

Test Information

Test Name DAMS FINAL CBT Total Questions 300

Test Type Examination

210
Total Marks 1200 Duration
minutes

Test Question    Language:- ENGLISH

1. Distal end of humerus develops from how many centres

a. 2

b. 3

c. 4

d. 5

Solution. (c) d.
Ref: Read the text below
Sol :
• Centers are :- capitulum with the lateral part of the trochlea, the medial part of the trochlea, and one for each
epicondyle. The centre for the medial epicondyle forms a separate epiphysis whereas The centres for the lateral
epicondyle, capitulum and trochlea fuse around puberty and form the composite epiphysis About the sixteenth or
seventeenth year, the lateral epicondyle and both portions of the articulating surface, having already joined, unite with
the body, and at the eighteenth year the medial epicondyle becomes joined to it.

Answer. c

2. Tympanic plexus is present on:

a. Anterior wall of middle ear.

b. Posterior wall of middle ear.

c. Medial wall of middle ear.

d. Lateral wall of middle ear.

Solution. (c) Medial wall of middle ear


Ref: Read the text below
Sol :

• In the tympanic cavity, the tympanic nerve (a branch of CN IX) divides into branches which, along with
sympathetic fibres from the carotid plexus, form the tympanic plexus.

• This plexus is located on the surface of the promontory (Medial wall of middle ear)

Answer. c

3. Which nerve lesion could produce the condition where stimulation of right cornea results in blinking of the left
eye but not the right eye?

a. Left trigeminal

b. Left facial

c. Right trigeminal

d. Right facial

Solution. (d) Right facial


Ref: Read the text below
Sol :
Answer. d

4. Atrophy of intrinsic muscles of hand, sensory deficit on medial side of forearm and hand, and diminished radial
pulse on turning the head on the affected side could be because of :

a. Carpal tunnel syndrome

b. Cervical rib

c. Enlarged axillary lymph nodes

d. Supracondylar fracture of humerus

Solution. (b) Cervical rib


Ref: Read the text below
Sol :
▪ Cervical ribs (present in upto 1% of people) articulate with the C7 vertebra and are clinically significant because
they may compress spinal nerves C8 and T1 or the inferior trunk of the brachial plexus supplying the upper limb. Tingling
and numbness may occur along the medial border of the forearm. They may also compress the subclavian artery, resulting
in ischemic muscle pain (caused by poor blood supply) in the upper limb.

Answer. b

5. Fibular collateral ligament is the continuation of:

a. Peroneus longus

b. Biceps femoris

c. Semimembranosus

d. Adductor magnus

Solution. (a) Peroneus longus


Ref: Read the text below
Sol :
• Fibular collateral ligament is a degenerated part of Peroneus Longus muscle.
• Oblique popliteal ligament is a continuation of semimembranosus muscle. It lies in the floor of popliteal fossa.
• The tibial collateral ligament represents the degenerated part of adductor magnus.
• The sacrotuberous ligament represents the degenerated part of the long head of biceps femoris.
• Sacrospinous ligament represents the degenerated part of Coccygeus muscle.

Answer. a

6. Identify the muscle :

a. Lateral cricoarytenoid

b. Posterior cricoarytenoid

c. Thyroepiglottis

d. Cricothyroid
Solution. (b) Posterior cricoarytenoid
Ref: Read the text below
Sol :

Answer. b

7. A 31-year-old woman complains of headache and dizziness after hitting a kitchen cabinet door with her head.
Her magnetic resonance imaging (MRI) scan and venogram show a large blood clot in the great cerebral vein
of Galen. The obstructed vein of the brain is a direct tributary of which of the following venous structures?

a. Emissary veins

b. Pterygoid venous plexus

c. Diploic veins

d. Dural venous sinuses

Solution. (d) Dural venous sinuses


Ref– Read the text below
Sol:
• The veins of the brain are direct tributaries of the dural venous sinuses.
• The emissary veins connect the dural venous sinuses with the veins of the scalp; the pterygoid venous plexus
communicates with the cavernous sinus through an emissary vein; the diploic veins lie in channels in the diploc of
the skull and communicate with the dural sinuses, the veins of the scalp, and the meningeal veins.

Answer. d

8. A 43-year-old man has new onset of difficulty with speaking. Examination reveals problems in elevating the
hyoid bone and floor of the mouth, secondary to paralysis of the posterior belly of the digastric muscle. Which
of the following nerves is most likely involved?

a. Accessory nerve

b. Trigeminal nerve

c. Ansa cervicalis

d. Facial nerve

Solution. -(d) Facial nerve


Ref– Read the text below
Sol:
• The digastric posterior belly is innervated by the facial nerve, whereas the digastric anterior belly is innervated by
the trigeminal nerve.
• The accessory nerve supplies the sternocleidomastoid and trapezius muscles. The ansa cervicalis innervates the
infrahyoid (or strap) muscles.
• The glossopharyngeal nerve supplies the stylopharyngeus muscle.

Answer. d

9. Clitoris develops from

a. Genital tubercle

b. Genital folds

c. Labio-scrotal swelling

d. Urogenital sinus

Solution. (a) Genital tubercle


Ref: Read the text below
Sol :
Answer. a

10. Medial meniscus is more prone to injury because:

a. It is semi lunar

b. Attached firmly to tibial collateral ligament

c. It is avascular

d. Connected to lateral meniscus by lateral ligament

Solution. -(b) Attached firmly to tibial collateral ligament


Ref: Read the text below
Sol :
• Medial meniscus is firmly attached to the capsule of knee joint & to the tibial collateral ligament.
• During any forceful movements it gets crushed between the two.
• The lateral meniscus is separated from the capsule of knee joint by the tendon of popliteus muscle.

Answer. b

11. In carpal tunnel syndrome which of the following occurs :

a. Loss of sensation over thenar eminence

b. Flexor digitorum superficialis paralysis

c. Flexor pollicis brevis paralysis

d. Adductor pollicis paralysis

Solution. (c) Flexor pollicis brevis paralysis


Ref: Read the text below
Sol :
• Flexor polices brevis is supplied by the median nerve in hand, after passing through the carpel tunnel.
• Flexor digitorum superficialis is supplied by the median nerve in the forearm.
• The superficial palmar branch crosses superficial to the flexor retinaculum & supplies the skin over the thenar
eminence.
• The adductor polices is supplied by the deep branch of ulnar nerve in the hand.

Answer. c

12. Portal Vein develops from which of these structures :

a. A

b. B

c. C

d. D
Solution. (c) C
Ref: Read the text below
Sol :
• Portal vein develops from vitelline vein (Right)

Answer. c

13. In the diagram shown below what is true about the structures piercing clavipectoral fascia ?

a. a. Thoracoacromial Trunk b. Cephalic vein c. Lateral Pectoral nerve

b. a. Thoracoarcromial Trunk b. Basilic vein c. Lateral Pectoral nerve

c. a. Lateral thoracic artery b. Basilic vein C. Lateral thoracic nerve

d. a. Lateral Thoracic b. cephalic vein C Lateral Pectoral nerve

Solution. (a) a. Thoracoacromial Trunk b. Cephalic vein c. Lateral Pectoral nerve


Ref: Read the text below
Sol :
➢ A. Thoracoacromial Trunk
➢ B. Cephalic vein
➢ C. Lateral Pectoral nerve

Answer. a

14. Cell lining of common bile duct is:

a. Stratified columnar

b. Stratified squamous

c. Simple cuboidal

d. Simple columnar

Solution. d. Simple columnar


Ref: Read the text below
Sol:
• The intrahepatic ducts, cystic duct, and the common bile duct are lined by a tall columnar epithelium.

Answer. d

15. Respiratory bronchiole proceed into:

a. Terminal bronchiole

b. Bronchi

c. Tertiary bronchiole

d. Alveolar duct

Solution. d. Alveolar duct


Ref: Read the text below
Sol :
• Bronchioles have a diameter smaller than 1 mm and lack cartilage and glands within their walls. Goblet cells (and cilia)
decrease in number and almost negligible at the levels of bronchioles (small lumen). Hyaline cartilage also is almost
non-existent at the levels of bronchioles. Epithelium gradually changes from pseudostratified columnar to simple columnar
to cuboidal to squamous.
• Alveolus is lined by type-I pneumocyte (simple squamous epithelium) for respiratory gas exchange. Type-II pneumocyte
is a cuboidal cell for surfactant secretion.
• Bronchial arteries (branches of descending thoracic aorta) supply till the level of respiratory bronchiole. They perfuse
the proximal air conducting pathways including tertiary and terminal bronchioles ane reach till the beginning of
respiratory unity. Pulmonary arteries alone vascularize the further distal pathways, including alveolar ducts and the
alveoli.

Answer. d

16. Nalorphine is used as an antidote for all of the following, except

a. Morphine

b. Pethidine

c. Pentazocine

d. Heroin

Solution. (c) Pentazocine


Ref:Read the text below
Sol:
- Nalorphine is a strong μ receptor antagonist, so can be used as an antidote for μ agonists like morphine,
pethidine and heroin.
- Nalorphine is also an agonist of κ receptors and pentazocine is also mainly a κ agonist, so nalorphine is not an
antidote for pentazocine.

Answer. c

17. Methemoglobinemia caused by

a. Procaine

b. Prilocaine

c. Bupivacaine

d. Lignocaine

Solution. (b) Prilocaine

Ref:Read the text below

Sol:

Morgan Clinical Anesthesiology, Fourth Edition

·      Metabolites of prilocaine (o-toluidine derivatives), which accumulate after large doses of drug (> 10 mg/kg),
convert hemoglobin to methemoglobin.

·      Neonates of mothers who have received prilocaine epidural anesthesia during labor and patients with limited
cardiopulmonary reserve are particularly susceptible to the alteration in oxygen transport.

·      Benzocaine, a common ingredient in local anesthetic sprays, can also cause methemoglobinemia

.       Treatment of significant methemoglobinemia includes intravenous administration of methylene blue (1–2 mg/kg of a
1% solution over 5 min). Methylene blue reduces methemoglobin (Fe3+) to hemoglobin (Fe2+).

Answer. b

18. The following are used for treatment of postoperative nausea and vomiting following squint surgery in children
except :

a. Ketamine

b. Ondansetron

c. Propofol

d. Dexamethasone
Solution. (a) Ketamine
Ref: Read the text below
Sol:
Drugs used for Postoperative nausea & vomiting following squint surgery.
(1) Propofol infusion plus nitrous oxide
(2) Transdermal scopolamine
(3) Ondansetron and Granisetron
(4) Droperidol
• Dexamethasone is used for the prevention of post-operative nausea-vomiting, especially chemotherapy induced nausea,
vomiting. It may be useful particularly when used in combination with ondanstron.
• No role of ketamine in the prevention of post-operative nausea - vomiting (ketamine increases the intragastric pressuer).
• Ketamine also increases intraocular pressure.

Answer. a

19. Which of the following is a contraindication for using a laryngeal mask airway ?

a. When inhalational anaesthesia is required

b. Large obstructive lesion in the oropharynx

c. Maintaining airway during difficult intubation

d. Emergency management of airway in failed intubation

Solution. (b) Large obstructive lesion in the oropharynx


Ref: Read the text below
Sol:
• Laryngeal Mask Airway is an alternative airway device used for anesthesia and airway support. It consists of an
inflatable silicone mask and rubber connecting tube. It is inserted blindly into the pharynx, forming a low-pressure seal
around the laryngeal inlet and permitting gentle positive pressure ventilation. All parts are latex-free.
• The Laryngeal Mask Airway is an appropriate airway choice when mask ventilation can be used but endotracheal
intubation is not necessary.
Contraindications:
Non-fasted patients
Morbidly obese patients
Obstructive or abnormal lesions of the oropharynx.

Answer. b

20. The anesthetic agent which can be used during labour for intrauterine fetal manipulations as it causes profound
uterine relaxation is:

a. Nitrous oxide

b. Desflurane

c. Midazolam

d. Ketamine

Solution. (b) Desflurane


Ref: Read the text below
Sol:
• Halogenated hydrocarbon anesthetics are potent relaxants of the uterine muscle.

Answer. b

21. Postdural puncture headache

a. Results from leakage of CSF into subdural space

b. Increased hydrostatic pressure in subarachnoid space

c. Relieved in upright posture

d. Triptans might be needed for treatment

Solution. (d) Triptans might be needed for treatment


Ref: Read the text below
Sol:

• Post dural puncture headache (PDPH) is a complication of puncture of the dura mater (one of the meninges that
surround the brain and spinal cord). This occurs inspinal anesthesia and lumbar puncture and may, accidentally,
occur in epidural anesthesia.
• PDPH typically occurs hours to days after puncture and presents with symptoms such as headache and nausea that
typically worsen when the patient assumes an upright posture.
• It is thought to result from a loss of cerebrospinal fluid into the epidural space. A
decreased hydrostatic pressure in the subarachnoid space then leads to traction to
the meninges with associated symptoms.
• The incidence of PDPH is higher with younger patients, complicated or repeated puncture, and use of large diameter
needles.Some patients require no other treatment than analgesics, caffeine, and bed rest. Other patients
might need triptan.
• However, persistent and severe PDPH may require an epidural blood patch. A small amount of the patient's blood
is injected into the epidural space near the site of the original puncture; the resulting blood clot then "patches" the
meningeal leak. The procedure carries the typical risks of any epidural puncture.

Answer. d

22. Conditions or drugs which causes rise in CVP include all except.:

a. IPPY

b. Heart failure

c. Nitroprusside

d. Valsalva manoeuvre.

Solution. (c) Nitroprusside


Ref: Read the text below
Sol:
CVP is increased in:
1. Congestive cardiac failure, constrictive pericarditis, cardiac tamponade.
2. Hypervolemia.
3. Increased venous pressure (e. g., by catecholamines).
4. Coughing and straining (Valsalva manoeuvre).
5. Positive pressure ventilation (lPPV and PEEP).
6. Pleural effusion hemothorax.
CVP is decreased in:
1. Shock and hypovolemia.
2 Vasodilation (sodium nitroprusside, nitroglycerine.

Answer. c

23. Which is not a coaxial circuit:

a. Lack

b. Bain

c. Penlon

d. Jackson Rees.

Solution. (d) Jackson Rees


Ref:Read the text below
Sol:
Coaxial system are systems with two tubing - one inner and one outer. Coaxial circuit are:
1. Lack.
2. Bain.
3. Penlon.
4. Humphry ADE.
5. Mera F

Answer. d

24. Which of the following agents is long-duration ester for a local anesthesia ?

a. Bupivacaine

b. Cocaine

c. Lidocaine

d. Tetracaine
Solution. (d) Tetracaine
Ref:Read the text below
Sol:
-Tetracaine is a long – duration ester local anesthetic.

Answer. d

25. The volatility of an anesthetic agent is directly proportional to lowering the flow in the portal vein. Portal flow is
maximally reduced by

a. Ether

b. Halothane

c. Isoflurane

d. Enflurane

Solution. (b) Halothane


Ref:Read the text below
Sol:
• Halothane impairs liver blood flow and oxygenation the most, perhaps explaining increased incidence with its use.
• Enflurane was shown to reduce hepatic blood flow less than halothane.

Answer. b

26. What is the approximate number of base pairs associated with a single nucleosome?

a. 146

b. 292

c. 73

d. 1460

Solution. (a) 146


Ref.: Read the text below
Sol :
• 1.75 turn of double strand DNA contains 146 base pair.
• Linker DNA contains 20 to 90 base pairs

Answer. a

27. Which of the following is NOT a hemoprotein?

a. Myoglobin

b. Cytochrome c

c. Catalase

d. Albumin

Solution. (d) Albumin


Ref.: Read the text below
Sol :
Following is the list of heme protein
Haemoglobin
Myoglobin
Catalase
Peroxidise
Nitric oxide synthase
Guanylyl cyclise

Answer. d

28. Heparin is a:

a. Glycoprotein

b. Glycosaminoglycan
c. Lipoprotein

d. Protein

Solution. (b) Glycosaminoglycan


Ref.: Read the text below
Sol :
• Heparin, the anticoagulant is a mucopolysaccharide (glycosaminoglycan).
• It binds to a specific cationic site of antithrombin, including a conformational change and promoting its binding
to thrombin as well as to its other substrates.
• This is the basis for the use of heparin to inhibit coagulation.

Answer. b

29. All of the following are true about NADPH except –

a. Produces ATP in RBC

b. Helps in reductive biosynthesis

c. Formed by glucose-6-phosphate dehydrogenase

d. helps in maintaining membrane structure of RBC

Solution. (a) Produces ATP in RBC


Ref.: Read the text below

Sol :
• NADPH never produces ATP by oxidative phosphorylation, it rather is used in reductive biosynthesis and
handling oxidative stress.

Answer. a

30. Essential pentosuria is due to defect in ?

a. HMP shunt pathway

b. Glycolysis

c. Gluconeogenesis

d. Uronic acid pathway

Solution. (d) Uronic acid pathway


Ref.: Read the text below
Sol :
• Xylitol dehydrogenase deficiency will lead to essential pentosuria,which results in excretion of L-Xylulose in the
urine.

Answer. d

31. In well fed state, acetyl Co A obtained from the diet is least used for the synthesis of ?

a. Palmitoyl CoA

b. Citrate

c. Acetoacetate

d. Oxaloacetate

Solution. (c) Acetoacetate


Ref.: Read the text below
Sol :
• In well fed state, acetyl Co A from diet is mainly used for TCA cycle or fatty acid synthesis.
• It is least used for formation of ketone body( acetoacetate).

Answer. c

32. In last couple of hours at the end of 48 hours of fasting the main source of energy is

a. Muscle glycogen
b. Liver glycogen

c. Acetoacetate

d. Nucleic acids

Solution. (c) Acetoacetate


Ref.: Read the text below
Sol :
• At the end of 48 hours, ketone body becomes the available source of energy.

Answer. c

33. The breakdown of one molecule of a C16 fully saturated fatty acid (palmitic acid) by β-oxidation lead to formation of:

a. 7 FADH2, 7NADH and 7 acetyl-CoA molecules

b. 8 FADH2, 8NADH and 7 acetyl-CoA molecules

c. 7 FADH2, 8NADH and 8 acetyl-CoA molecules

d. 7 FADH2, 7NADH and 8 acetyl-CoA molecules

Solution. (d) 7 FADH2, 7NADH and 8 acetyl-CoA molecules

Ref.: Read the text below


Sol :
• In total 7 cycle 16 carbon fatty acid is oxidized resulting in 7FADH2, 7NADH and 8 acetyl-CoA molecules production
which forms 108 total ATP and 106 net ATP.

Answer. d

34. α-Linolenic acid is considered to be nutritionally essential in humans BECAUSE:

a. It is an ω3 fatty acid.

b. It contains three double bonds.

c. In humans double bonds cannot be introduced into fatty acids beyond the Δ9 position.

d. In humans double bonds cannot be introduced into fatty acids beyond the Δ12 position.

Solution. (c) In humans double bonds cannot be introduced into fatty acids beyond the Δ9 position.
Ref.: Read the text below
Sol :
• α-Linolenic acid and linoleic acids are considered to be nutritionally essential in humans because double bonds
cannot be introduced into fatty acids chain beyond the Δ9 position due to lack of desaturase enzyme beyond Δ9.

Answer. c

35. Which of the following amino acid is an iminoacid :

a. Proline

b. Hisitidine

c. Arginine

d. Lysine

Solution. (a) Proline


Ref: Read the text below
Sol:
• Proline: proline has got imino group instead of amino group at alpha carbon position. This imino group form
pyrrolidine ring with the side chain.
• Proline and hydroxyproline gives yellow colour in ninhydrin test compare to all other amino acids which gives
purple colour.

Answer. a

36. Protein is purified using ammonium sulfate by


a. Salting out

b. Ion exchange chromatography

c. Mass chromatography

d. Molecular size exclusion

Solution. (a) Salting out


Ref.: Read the text below
Sol :
• Salting out: on adding the ammonium sulphate in a protein solution, protein undergoes precipitation as ammonium
sulphate attracts the shell of hydration to dissolve itself.
• As a general rule higher the molecular weight of the protein lesser salt is required for precipitation.
• Globulin is precipitated at half saturation and albumin is saturated at full saturation.

Answer. a

37. 1stAmino acid Selenocysteine is derived from

a. Cysteine

b. Cystine

c. Serine

d. Alanine

Solution. -(c) Serine


Ref.: Read the text below
Sol :
• Serine: Oxygen atom of serine hydroxyl group is replaced by selenium by selenocysteine synthetase enzyme. So
selenocysteine is derived from serine.

Answer. c

38. Nicotinamide is derived from:

a. Histidine

b. Methionine

c. Tryptophan

d. Phenylalanine

Solution. (c) Tryptophan


Ref.: Read the text below
Sol :
• In a major catabolic route tryptophan is forming nicotinamide.
• This pathway is also known as kynurenine pathway as it needs kynureninase enzyme.

Answer. c

39. Beta oxidation of fatty acids requires :

a. FAD and NADP

b. FADP and NADP

c. FAD and NAD

d. FADH and NAD

Solution. (c) FAD and NAD


Ref.: Read the text below
Sol :
• During beta oxidation, acyl Co A dehydrogenase enzyme needs FAD as a coenzyme and beta hydroxyl acyl Co A
dehydrogenase enzyme needs NAD as a coenzyme.

Answer. c

40. Citrate used in fatty acid synthesis via which enzyme


a. Citrate Synthase

b. ATP citrate lyase

c. Aconitase.

d. Malic enzyme

Solution. (b) ATP citrate lyase


Ref.: Read the text below
Sol :
• Action of ATP citrate lyase provides acetyl co A in the cytosol of the cell , which is then utilized for fatty acid
synthesis

Answer. b

41. “Wave III” in the given Fig is related with which of the following anatomical site :

a. Eighth nerve (proximal segment)

b. Eighth nerve (distal segment)

c. Cochlear nuclei.

d. Inferior Colliculus.

Solution. (c) Cochlear nuclei


Reference – Read the text below
Sol:
ABR Peaks: Each peak may correlate to an anatomical location
• I–II: Eighth nerve (distal and proximal segments)
• III: Cochlear nuclei
• IV: Olive (superior)
• V: Lateral lemniscus (largest wave)
• VI–VII: Inferior Colliculus
Only waves I, III, and V are observed at birth, I–V is prolonged at birth

Answer. c

42. Resonance Frequencies are natural frequencies that vibrate a mass with the least amount of force. Resonance
frequency for Concha is

a. 800–1600 Hz

b. 800 Hz.

c. 4000–6000 Hz

d. 500–2000 Hz

Solution. (c) 4000–6000 Hz


Sol:
Resonance Frequencies: natural frequencies that vibrate a mass with the least amount of force
• Concha: 4000–6000 Hz
• EAC: 2000–3000 Hz (primary frequencies of speech)
• TM: 800–1600 Hz
• Ossicles: 500–2000 Hz
• Middle Ear: 800 Hz

Answer. c

43. In case of unilateral superior laryngeal nerve paralysis, posterior commissure points to :-

a. Paralyzed side

b. Normal side

c. Midline

d. None

Solution. (a) Paralyzed side.


Sol:
1. PPP rule (Posterior commissure Points to Paralyzed side in unilateral superior laryngeal nerve paralysis)

Answer. a

44. In the internal auditory canal, the vertical crest (Bill’s bar) marks a plane between :

a. Superior and inferior vestibular nerves

b. Facial and cochlear nerves

c. Inferior vestibular and singular nerves

d. Superior vestibular and facial nerves

Solution. (d) Superior vestibular and facial nerves


Reference – Otolaryngology by R.Pasha --358
Sol:
• The lateral wall of the IAC is the lamina cribrosa, and the superior fundus is divide vertically by Bill’s bar, a
triangular strut of bone seen only on axial images.
• Facial nerve transverses in the anterior superior quadrant of the IAC separated by the falciform crest inferiorly and
Bill’s bar posteriorly.
Other quadrants
• Superior vestibular nerve [superior posterior]
• Inferior vestibular nerve [inferior posterior]
• The cochlear nerve [inferior anterior]

Answer. d

45. The earliest nerve to be involved in acoustic neuroma is.

a. Vth nerve

b. VIIth nerve

c. X1 nerve

d. X11 nerve

Solution. : (a) Vth nerve


Sol:
                  Cranial nerve involvement in acoustic neuroma.

Vth nerve
• This is the earliest nerve to be involved
• There is reduced corneal sensitivity, numbness or par aesthesia of face.

Answer. a

46. Penetration of which structure allows infection to spread from the paranasal sinuses to the orbit?

a. Basal lamella

b. Fovea ethmoidalis
c. Uncinate process

d. Lamina papyracea

Solution. (d) Lamina papyracea


Reference – Otolaryngology by R.Pasha --5
Sol:
Lamina Papyracea:
• Lateral thin bony wall of the ethmoid sinus, separates orbit from ethmoid cells as a part of the medial orbital wall

Computerized axial tomography (CAT) radiograph of the nose.Coronal section view of the osteomeatal unit.
Fovea Ethmoidalis:
• Roof of ethmoid sinus
Ground (Basal) Lamella:
• Posterior bony insertion of the middleturbinate which separates anterior and posterior ethmoid cells; posterior extension
of the middle turbinate
Uncinate Process:
• Sickle-shaped thin bone part of the ethmoid bone, covered by mucoperiosteum, medial to the ethmoidinfundibulum and
lateral to the middle turbinate(derived from the second ethmoidal turbinal)

Answer. d

47. Tympanic plexus is formed by :

a. Spinal assessory nerve and sympathetic fibres.

b. Vagus nerve and sympathetic fibres.

c. Glossopharyngeal nerve and sympathetic fibres.

d. Arnold’s and sympathetic fibres.

Solution. (c) Glossopharyngeal nerve and sympathetic fibres.


Sol:
• Jacobson’s nerve, with Arnold’s nerve forms tympanic plexus on the promontory.
• Arnold’s nerve: auricular branch of the vagus nerve, traverses the tympomastoid fissure to innervte the posterior
aspect of the EAC auriculotemporal nerve
(sympathetic fibers)

Answer. c

48. Most common site of ‘squamous cell carcinoma’ nasal cavity is :

a. Superior turbinate.

b. Inferior turbinate.

c. Lateral wall

d. Middle meatus

Solution. (c) Lateral wall


Sol:
                                   NASAL CAVITY
• Most common tumour: squamous cell carcinoma
• Most common site of squamous cell carcinoma: Lateral wall
Answer. c

49. Abscess shown in the Fig spares :

a. CN IX

b. CN X

c. CN XI

d. CN VII

Solution. (d) CN VII


Sol:
• The parapharyngeal space may be divided into two compartments on the basis of its relationship to the styloid
process or, more precisely, to the tensor-vascular-styloid fascia.
• The importance of the parapharyngeal space also lies in its relationship with the other spaces of the neck. The
masticator and parotid spaces are located laterally, the pharyngeal mucosal space is located medially, and the
retropharyngeal space is located posteromedially.
• The contents of the prestyloid compartment include the minor or ectopic salivary gland, branches of the mandibular
division of the trigeminal nerve, internal maxillary artery, ascending
pharyngeal artery, and pharyngeal venous plexus, whereas those of the poststyloid compartment include the internal
carotid artery, internal jugular vein, cranial nerves IX-XII, cervical sympathetic chain, and glomus bodies.
• The facial nerve (CN VII) has only a brief course through the poststyloid space as it exits the skull via the
stylomastoid foramen

Answer. d

50. Elliptical space between the cords is seen in cases of weakness of :

a. Thyroarytenoid.

b. Interarytenoid.

c. Both thyroarytenoid and interarytenoid

d. None

Solution. (a) Thyroarytenoid.


Sol:
Phonasthenia: Weakness of voice.
• Elliptical space between the cords : Thyroarytenoid
• Triangular gap near the posterior commissure: Interarytenoid.
• Key – Hole appearance of glottis.

Answer. a

51. child with ambiguous genitalia with undescended testes on one side and mullerian structures on other side most likely
has which of the following

a. Refeinstein syndrome

b. Male psuedohermaphrodite
c. Mixed gonadal dysgenesis

d. True hermaphrodite

Solution. C
Explanation
Mullerian structures on one side means there was no exposure to MIS on that side which means no testes on that side,
while there is testes on other side with ambiguous genitalia is typical of mixed gonadal dysgenesis.

Answer. c

52. Cervical cerclage is indicated in all the following conditions except

a. Twin pregnancy

b. Cervical length less than 2.5 cm in second trimester

c. History of mid trimester loss with painless dilatation of cervix

d. Cervical dilatation of 3cm at 20 weeks

Solution. A
explanation
Cervical cerclage is not recommended as prophylaxis for twin pregnancy as it has failed to prove as an effective method
for prevention of preterm birth and perinatal mortality.

While all others are indications of cerclage.

Answer. a

53. What is the most likely cause of infertility in a patient who has oligomenorrhoea and normal FSH and positive
progesterone withdrawal test

a. Premature ovarian failure

b. Anovulation

c. Ashermans syndrome

d. Hypothalamic amenorrhoea

Solution. B
explanation
Normal FSH and a progesterone challenge test is reflecting towards progesterone deficiency and hence the most likely
cause is anovulation. In premature ovarian failure FSH should be high, in ashermans the withdrawal test should be
negative and in hypothalamic amenorrhoea again the progesterone withdrawal test is negative .

Answer. b

54. Regarding downs syndrome incorrect is

a. Cystic hygroma in first trimester has high risk

b. Nuchal translucency is done at 16-18 weeks

c. Hypoplastic nasal bone is an important soft marker

d. Recurrence risk is 1%

Solution. B
Explanation
Nuchal translucency is a ultrasound marker in first trimester and the ideal time to look for it is between 11 weeks and 13
weeks and 6 days, values more than or equal to 3mm is a positive test. Recurrence risk is 1% and absent or hypolplastic
nasal bone is an important soft marker along with increased nuchal fold thickness

Answer. b

55. In boys with precocious puberty with prepubrtal LH levels which of the following is the investigation of choice

a. CT scan of adrenal

b. MRI brain
c. Visual field testing

d. FSH level

Solution. A
explanation
In precocious puberty prepubertal level of gonadotropins indicate peripheral excess of androgens which is supressing the
LH and FSH hence we must look for evaluation of adrenal gland. MRI brain should be done when you suspect a central
cause and then the gonadotropin levels will be high.

Answer. a

56. A 29 year old woman at 38 weeks with three previous normal deliveries, has a transverse lie conformed on ultrasound.
There is no obstruction in the pelvis. Which of the following is the management of this patient

a. No intervention

b. Internal version with breech extraction

c. Caesarean section

d. ECV

Solution. D
Explanation
Transverse lie at term is a candidate for external cephalic version provided the prerequisites are met. The question says
she has had three normal deliveries and there is no obstruction in pelvis which indicates no problem in vaginal delivery.
For all non cephalic presentations we must try ECV. Internal version with breech extraction is done for second fetus in
transverse lie in twin pregnancy.

Answer. d

57. What is the average liquefaction time for semen sample

a. 10-15 mins

b. 20-30 mins

c. 30-45 mins

d. 45-60 mins

Solution. B
Liquefaction time affects the fertility status and hence is impotant. The average time is 20-30 mins. Time of more than 30
mins affects fertility as it impairs the sperms ability to reach the ovum. Infection is the most common cause of prolonged
liquefaction time.

Before semen analysis there must be abstinence of 2-7 days and abnormal reports should be confirmed on a second
sample taken 2-4 weeks apart.

Answer. b

58. which of the following does not usually respond to medical management of endometriosis

a. Dysmenorrhoea

b. infertility

c. Chocolate cyst

d. Bowel symptoms

Solution. C
Explanation
A chocolate cyst or endometrioma does not usually respond to medical management and needs surgical treatment. The
criteria for surgical treatment is size greater than 5cm. medical management for bowel endometriosis is effective in many
women and only when not responding we go ahead with surgical management.

Answer. c

59. which of the following is not a risk factor for trophoblastic disease
a. Age more than 40

b. Asian ethnicity

c. Previous molar pregnancy

d. Diet deficient in carbohydrates

Solution. D
Explanation
The following are the risk factors for trophoblastic disease
1) Prior molar pregnancy
2) Age more than 35 years risk is twofold and 7.5 fold for women more than 40
3) Asian origin
4) Women younger than 20 years
5) Prior miscarriage
6) Diet deficient in carotene and vitamin A
7) Blood types A or AB are at slightly higher risk
8) Cigarette smoking

Answer. d

60. All are true about sonographic appearance of complete molar pregnancy except

a. Bilateral theca lutein cyst

b. Identification is difficult on ultrasound

c. Blighted ovum is a differential diagnosis

d. Echogenic complex mass on ultrasound

Solution. B

Answer. b

61. Intrauterine insemination is indicated in all except

a. Tubal factor infertility

b. Unexplained infertility

c. endometriosis

d. male factor infertility

Solution. A
Explanation
Tubal factor infertitliy cannot be managed by IUI as pregnancy with IUI requires patent fallopian tubes
Indications of IUI
1) mild male factor infertility
2) antisperm antibodies
3) sexual dysfunction
4) unexplained infertility
5) minimal and mild endometriosis related infertility
6) pregnancy for single women

Answer. a

62. Passage of decidual cast is characteristic of

a. complete abortion

b. tubal abortion

c. missed abortion

d. incomplete abortion

Solution. B
The decidual cast is the lining of the pregnant uterus, it begins to develop when the woman gets pregnant under the effect
of the hormone progesterone. This lining is shed off in ectopic pregnancy and it comes out in the shape of uterine cavity.
The passage of this cast is sometimes mistaken as miscarriage but is shed off as the amount of progesterone is less and is
unable to maintain the endometrium.
It may also sometimes be seen in women taking combined oral contraceptives and women on DMPA .
Answer. b

63. which of the following is not true for the management of PPH

a. Use of isotonic crystalloids is preferred over colloids for resuscitation

b. Tranexamic acid can be used if bleeding is not responding to oxytocin

c. Bleeding after artery ligation is managed by hysterectomy

d. If medical methods fail to control bleeding then uterine packing can be done

Solution. D
Uterine packing is not recommended for management of PPH what can be done is balloon tamponade .
The use of tranexamic acid is recommended for treatment if oxytocin or other uterotonic drugs fail to stop bleeding or if it
is felt that bleeding may be partly due to trauma. The other two statements are true

Answer. d

64. Name the procedure shown below

a. Parkland

b. Pomeroy

c. Madlener

d. Band application

Solution. A
Explanation
This is the parkland technique where a lo are ligated separately with plain catgut.

Answer. a

65. the following Hsg image was seen in evaluation of infertility in a 35 year old lady, what is the appearance
a. Tobacco pouch

b. Golf club

c. hydrosalpinx

d. lead pipe

Solution. B
Explanation this is the characteristic GOLF CLUB APPEARANCE OF FALLOPIAN TUBE SEEN in genital tuberculosis

Answer. b

66. all the following are predictors of preeclampsia except

a. notching in uterine artery doppler

b. decreased level of VEGF

c. increased level of serum uric acid

d. increased level of sFlt-1

Solution. C
Serum uric acid is raised in preeclampsia b preeclampsia. The vasoprssors like sFlt1 and sEng are raised while
vasodilators like VEGF and PlGF are decreased.

Answer. c

67. All the following can be done for management of vault prolapse except

a. sacrocolpopexy

b. colposuspension

c. uterosacral suspension

d. Le Fortes

Solution. B
Colposuspension is a surgery for stress urinary incontinence and not for vault prolapse. The best surgery for vault
prolapse is sacrocolpopexy.

Uterosacral suspension can be used for both prophylaxis as well as treatment for vault prolapse Le fortes can also be used
as it obliterates the vaginal opening by creating two small cavities through which the vault cannot fall out

Answer. b

68. All the following are physiological in second trimester of pregnancy except

a. Decrease in fasting plasma glucose


b. Increase in peripheral resistance

c. Increase in cardiac output

d. Increase in GFR

Solution. B
The peripheral vascular resistance falls maximally in the second trimester between 24-26 weeks this is because of fall in
both DBP and SBP.

Cardiac output starts increasing form 5 weeks and reaches a maximum value between 28-32 weeks The GFR increases in
pregnancy by 50%

Because of increasing insulin resistance the fasting glucose levels are low while postprandial levels are high.

Answer. b

69. which of the following is true about gonadotropins

a. High levels in reproductive age group

b. Cyclical release after menopause

c. Raised upto 18 months after birth

d. Receptors are intranuclear

Solution. C
Explanation
After birth the steroid levels in the child fall precipitously due to loss of maternal and placental hormones allowing the
newborns HPO axis to escape their suppressive effects. The characteristic pulsatile pattern of hypothalamic GnRH
secretion emerges and serum gonadotropin concentration rise rise again promptlywitha striking sex difference, FSH rises
to a greater extent in females and LH rises more in males. Gonadotropin and steroid levels peak at about 3-6 months in
boys and 12- 18months in girls after which they begin to fall to come to prepubertal levels

Answer. c

70. what is the dosing schedule of centchroman?

a. 15mg twice a week

b. 30 mg once a week

c. 30 mg twice a week for 3 months then once a week

d. 15mg twice a week for 3 months and then once a week

Solution. C
Centchroman which was earlier marketed as saheli and is now made available by government of India as Chhaya, it is a
non hormonalnon steroidal contraceptive pill.
It is once a week contraceptive pill. But to be taken as 30 mg twice a week for first 3 months and then once a week. The
pill to be started on first day of menstrual cycle.
It is a reversible method, it is sfae even in breast feeding women even immediately after childbirth. Return of fertility is
prompt after stopping of pill.

Answer. c

71. the following ovarian tumour was seen in a 50 year old female associated with ascites and pleural effusion. What is
the likely diagnosis
a. Granulosa cell tumour

b. Fibroma

c. Thecoma

d. Mucinous ovarian tumour

Solution. B
Explanation
This is a case of meigs syndrome with the triad of fibroma, ascites and pleural effusion. The management is removal of
fibroma and the ascites and effusion will resolve on its own. Fibroma is a type of stromal ovarian tumour.

Answer. b

72. a 55 year old lady comes to gyane OPD with complaints of two episodes of postmenopausal bleeding, all the following
investigations should be done for her evaluation except

a. Transvaginal ultrasound

b. PAPs

c. Endocervical curettage

d. Endometrial aspiration

Solution. C
The first investigation to be done is TVS to look for endometrial thickness and IOC is endometrial biopsy or endometrial
aspiration. PAP smear should also be included as one of the investigations whether to assess he endocervix could be
decided based on report of PAPsmear.

Answer. c

73. which of the following is incorrect about the device shown


a. Has higher expulsion rate than CuT 380 A

b. Has a life span of 5 years

c. Acts by inhibition of fertilisation

d. Less risk of infection compared to CuT380

Solution. A
Explanation
This is multiload 375 device, it comes preloaded on a inserter and therefore the post insertion infection rates are less. Also
because of the bent arms the expulsion rate is less as compared to copper t 380A. just like all other intrauterine devices it
acts by inhibition o inhibition of implantation.

Answer. a

74. Regardingmalpresentations incorrect is

a. Face presentation is contraindication for forceps

b. Occipetoposterior may occur if vertex remains deflexed on entering the pelvis

c. Risk of cord prolapse is highest with transverse lie

d. Incidence of breech at term is 3%

Solution. A
Explanation
Face presentation is not a contraindication for forceps it can be used in mentoanterior positions Risk of cord prolapse is
highest in in transverse lie and among breech it is highest in footling breech.The most common cause of breech is
prematurity and the incidence at 28 weeks is about 25% while at term it is 3%

Answer. a

75. Which leopold’smaneuver is shown in the picture


a. Second

b. Fourth

c. First

d. Third

Solution. B
Explanation
This is the fourth leopoldsmaneuver
The first is FUNDAL grip to know what part occupies the fundus
Second is the lateral grip to know where the fetal back is
Third grip is to know what part lies above the inlet, this is also called as first pelvic grip
Fourth grip is to know the location of the brow, it is also called the second pelvic grip

Answer. b

76. The diagnosis of allergic contact dermatitis is confirmed with

a. Patch test

b. Prick test

c. Radioallergen sorbent test

d. IgE level estimation

Solution. 1-(a) Patch test


Ref: Bolognia Dermatology, 3rd edition; p 236.
Sol:
● Allergic contact dermatitis is confirmed with patch test.
● Patch test is aimed for identification of allergens by applying them on skin under
occlusion for 48 hours to elicit a delayed type hypersensitivity.
● IgE and radioallergen sorbent test as well as prick test are used for acute type I
hypersensitivity, whereas allergic contact dermatitis is a type IV hypersensitivity.

Answer. a

77. The drug not used in psoriasis is

a. Psoralens

b. Chloroquine

c. Methotrexate
d. Coaltar

Solution. (b) Chloroquine

Reference – Read the text below

Sol:

●  Wide spread psoriasis: by Ultraviolet β spectrum alone or along with coaltar (Geockerman regimen) or Anthranil
(Ingram regimen) Methotrexate Therapy; Methortrexate is drug of choice in very severe form of psoriatic arthritis.

●  Psoralens & Ultraviolet-A phototherapy (PUVA); is successful in clearing & delaying recurrence of Chronic
Psoriasis.

●  Etritinate, & its metabolite Acitretin, are Retinoids (Vitamin A derivatives); useful for severe Resistant Psoriasis.

●  Systemic drugs useful in Psoriasis: Hydroxyurea

●  Antimalarial like chloroquine, beta blockers, salycilates, lithium and ibuprofen are known provocative factors for
psoriasis.

Answer. b

78. The antigen in pemphigus vulgaris

a. Desmocollin

b. Desmoglein 1

c. Desmoplakin

d. Desmoglein 3

Solution. (d) Desmoglein 3


Reference – Read the text below
Sol:
● In pemphigus vulgaris, IgG antibodies are directed against desmoglein 3 (sometimes Dsg1 also). In pemphigus
foliaceus antibodies are directed against desmoglein 1.
● In paraneoplastic pemphigus, antibodies are directed against many antigens including desmoplakin. In endemic
pemphigus (fogo selvagem or Brazilian pemphigus), antibodies are directed against desmoglein 1. Desmocollin
is the target in subcorneal pustular dermatosis type of IgA pemphigus.

Answer. d

79. Monilethrix is

a. Regular beading of hair

b. Irregular beading of hair

c. Knotted hair

d. Broken hair.

Solution. (a) Regular beading of hair


Reference – Read the text below
Sol

● Regular beading of hair is called monilethrix.


● It is due to alternate zones of spindle – like thickening and thinning at 1 mm apart. Breaks develop at internodal
junction, where cortex and cuticle are defective.
● Nail and teeth defects, growth retardation, cataracts may be associated.
● Retinoids are effective in some cases.

Answer. a

80. The obstruction and level of escape of sweat in miliaria rubra is

a. Stratum corneum

b. Stratum lucidum

c. Stratum spinosum

d. Dermoepidermal junction
Solution. (c) Stratum spinosum

Ref: Andrews’ Diseases of The Skin,11th edition; p 19-20.

Sol:

●  Miliaria, the retention of sweat as a result of occlusion of eccrine sweat ducts, produces an eruption that is common in
hot, humid climates, such as in the tropics and during the hot summer months in temperate climates.

●  The occlusion prevents normal secretion from the sweat glands, and eventually pressure causes rupture of the
sweat gland or duct at different levels.

●  The escape of sweat into the adjacent tissue produces miliaria.

●  Depending on the level of the injury to the sweat gland or duct, several different forms are recognized.

●  Miliaria crystallinais characterized by small, clear, non itchy, superficial vesicles  due to leakage in stratum corneum.

●  The lesions of miliaria rubra (prickly heat) appear as discrete, extremely pruritic, erythematous papulovesicles
accompanied by a  sensation of prickling, burning, or tingling.

●  The site of injury and sweat escape is in the prickle cell layer (stratum spinosum), where spongiosis is produced.

●  Miliaria profunda has a blockage at or below the dermoepidermal junction.

Answer. c

81. Which of these biologic is not likely to be much useful in psoriasis

a. Secukinumab

b. Ustekinumab

c. Etanercept

d. Rituximab

Solution. (d) Rituximab


Ref: Bolognia Dermatology, 3rd edition; p 152, 53.
Sol:

●  Rituximab is anti-CD20 Ab.Thus its target is B cells and hence it is useful where immunoglobulins are involved.

●  Psoriasis is cell mediated mainly by TH17 and Th1 cells.

●  Etanercept, infliximab, adalimumab are classic TNF alpha inhibitorsused in psoriasis.

●  Ustekinumab and secukinumabwork on IL23 and IL17 respectively and hence used in psoriasis.

●  Rituximab in dermatology is used for immuno bullous diseases like pemphigus.

Answer. d

82. Erythema nodosum is type of

a. Panniculitis

b. Derrmatitis

c. Pruritus

d. Myositis

Solution. (a) Panniculitis


Ref: Andrews’ Diseases of The Skin,11th edition; p 143.
Sol:
● Eythema nodosum is a classic panniculitis that is inflammation of subcutaneous fat.
● It is septal panniculitis by histology and may be associated with drugs like OCP,
sulfonamides etc., pregnancy, TB, streptococcal infections, other infections including histoplasmosis,
coccidiodocomycosis, sarcoidosis, Behcet’s syndrome etc.

Answer. a

83. Which is correctly matched

a. Nikolsky sign: Bullous pemphigoid


b. Auspitz sign: Lupus vulgaris

c. Apple jelly nodules: Psoriasis

d. Dimpling sign: Dermatofibroma

Solution. (d) Dimpling sign: Dermatofibroma

Ref:Read the text below

Sol:

Nikolsky sign is negative in bullous pemphigoid and positive in pemphigus. Auspitz sign is seen in psoriasis while apple
jelly nodules are seen on diascopy in lupus vulgaris. Dimpling sign is typical of dermatofibroma.  A list of cutaneous signs
is given below:

a.Diascopy (VITROPRESSION) consists of pressing a transparent slide over a skin lesion. Examiner will find this of
special value to distinguish erythema or purpura. It is useful to detect the glassy yellow-brown appearance of papules in
sarcoidosis, tuberculosis(Apple Jelly Nodules) and other granuloma.

b.Darier's sign is positive when a brown macular or papular lesion of urticaria pigmentosa (cutaneous mastocytosis)
becomes palpable wheal after being rubbed with the blunt end of an instrument.

c.Grattage test & Auspitz’s sign is positive when slight scratching or curetting of a scaly lesion reveals initially fine
candle wax scales followed by red Berkley’s membrane; which then gets removed to reveal punctate bleeding
points(Auspitz's sign) within the lesion which suggests of psoriasis.

d.Nikolsky's sign is positive when a new blister is generated with ease by applying shearing force to skin or epidermis is
dislodged ( pemphigus, SSSS, TEN)

e.Asboe Hansen (Bulla spread) sign: Spread of bulla to normal skin by vertical pressure over in Pemphigus (~ to
Nikolsky’s)

f.  Carpet Tack Sign: or carpet en tack sign: removal of adherent scales in DLE reveals downward projection of scales
which are follicular plugs.

g.Chandelier’s sign: Pelvic Inflammatory Disease in women (intense pain on pelvic examination makes the patient leap
towards the chandelier.

h.Antenna sign: seen in Keratosis Pilaris.

i.   Buttonholing sign: Neurofibroma (with central vertical pressure the lesion disappears under the skin)

j.   Dimpling sign: dermatofibroma and dermatofibrosarcoma protuberans.

k.   Koebeners’ Phenomenon: Spread of lesions of same morphology (isomorphic phenomenon) at the site of trauma.
Characteristically seen in Lichen planus, psoriasis, vitiligo and by Auto-inoculation in Verruca, Molluscum
contagiosusm etc.

Answer. d

84. V-shaped nick at the free margin of nail is seen in

a. Hailey-hailey disease

b. Darier’s disease

c. Hypoalbuminemia

d. Cirrhosis

Solution. (b) Darier’s disease


Ref:Read the text below
Sol:
Important nail changes
● HALF AND HALF NAILS: Proximal portion is white and distal portion is brown with a sharp demarcating line. The
pigmentation is a result of melanin deposition (and
capillary proliferation). Seen in renal diseases with azotemia.
● TERRY’S NAILS: Distal 1-2 mm of nail is pink and the rest is white. Seen in cirrhosis, CHF, Diabetes.
● MEES’ LINES: Single or multiple white transverse bands on nail seen in chronic
arsenic poisoning, septicemia, chemotherapy, renal failure.
● MUEHRECKE’S NAILS: Narrow white bands occurring in pairs seen in chronic
hypoalbuminemia.
● LONGITUDINAL WHITE LINES WITH V-SHAPED NICKS AT FREE MARGIN OF NAILS: Darier’s disease.
● NAIL-PATELLA SYNDROME: Many anomalies of nail with hypoplastic patella.

Answer. b

85. An elderly patient has developed a slowly enlarging lesion as shown in the image. Likely diagnosis is
a. Squamous cell carcinoma

b. Melanoma

c. Cutaneous T cell lymphoma

d. Basal cell carcinoma

Solution. (d) Basal cell carcinoma


Ref:Read the text below
Sol:

● Chronic ulcer with rolled borders is typical of basal cell carcinoma.

Answer. d

86. Alkaline pyrogallol test helps to differentiate between

a. Ante-mortem and post-mortem burns

b. Air embolism and putrefactive gases

c. Ante-mortem and post-mortem clots

d. Acid and alkali poisoning

Solution. (b) Air embolism and putrefactive gases


Ref: The Essentials of Forensic Medicine & Toxicology, 34 th edition, Dr. K.S. Narayan Reddy,
Page No. 104
Sol:
Alkaline Pyrogallol test:
• This test helps to differentiate air embolism from putrefactive gases.4 mL of alkaline pyrogallol is taken in a
10mL syringe and air is aspirated from right side of the heart.
• The needle is removed and replaced by a stopper.
• If air (Oxygen) is present, the mixture turns brown on shaking.
• In a second syringe, some air is introduced by the atmosphere and the test is repeated as a control.
• In case of putrefactive gases, the colour doesn’t change.

Answer. b

87. Warning issued to a registered medical practitioner for

a. Unethical Practice

b. Wrong diagnosis

c. Wrong treatment

d. None of the above

Solution. (a) Unethical practice


Ref: The Essentials of Forensic Medicine & Toxicology, 34 th edition, Dr. K.S. Narayan Reddy,
Page No. 26
Sol:
Warning:
• MCI can issue Warning containing certain practices which are regarded as falling within the meaning of the term
“Professional Misconduct”.

Answer. a
88. Study the given image, Principle shown is used for:

a. Age estimation

b. To detect poisoning

c. Detecting crime

d. To find time since death.

Solution. -(c) Detecting crime


Ref: Read the text below
Sol:
• Locard's exchange principle holds that the perpetrator of a crime will bring something into the crime scene and leave
with something from it, and that both can be used as forensic evidence.

Answer. c

89. Large irregular entrance wound may be seen in

a. Contact shot

b. Close shot

c. Distant shot

d. Tandem bullet

Solution. (a) Contact shot  

Ref: The Essentials of Forensic Medicine & Toxicology, 34 th edition, Dr. K.S. Narayan Reddy, Page No. 209

Sol:

·   Where a thin layer of skin overlies bone, as in the head, the gases expand between the  skin and the outer table of
the skull, lifting up and ballooning out the skin.

·   If the stretching exceeds the elasticity of the skin, it will tear.

·   These tears radiate from the entrance, producing a stellate or cruciform appearing wound of entrance.

Answer. a

90. Cause of death in Commotiocordis is

a. Vagal inhibition

b. Cardiac tamponade

c. Cardiac rupture

d. Ventricular fibrillation

Solution. -(d) Ventricular fibrillation


Ref: The Essentials of Forensic Medicine & Toxicology, 34 th edition, Dr. K.S. Narayan Reddy,
Page No. 252
Sol:
• Commotiocordis typically involves young (8-18 years; average age is 15), predominantly male, athletes in whom a
sudden, blunt, non-penetrating and innocuous-appearing trauma to the anterior chest results in cardiac arrest
and sudden death from ventricular fibrillation.
Answer. d

91. The causative electrode of electric injury can be identified by

a. Acro reaction

b. Current pearl

c. Bone pearls

d. Wax drippings

Solution. (a) Acro reaction


Ref: The Essentials of Forensic Medicine & Toxicology, 34th edition, Dr. K.S. Narayan Reddy,
Page No. 375
Sol:
• Small balls of molten metal, derived from the electrode, may be carried deep into the tissues(Current Pearls); this
can be detected by a micro-chemical test known as, “Acro-Reaction Test”.
• Helps to identify the wound of entry.

Answer. a

92. All the following give rise to dimness of cornea before death EXCEPT

a. Uraemia

b. Narcotic Poisoning

c. Wasting Disease

d. Carbon Monoxide poisoning

Solution. (d) Carbon Monoxide poisoning


Ref: The Essentials of Forensic Medicine & Toxicology, 34 th edition, Dr. K.S. Narayan Reddy,
Page No. 145
Sol:
• Glistening of cornea remains even after death in case of CO and Cyanide poisoning.
• Dimness of cornea before death occurs in Cholera, Uraemia, Narcotic poisoning and wasting disease.

Answer. d

93. Deep inspiration above the water level may cause

a. Late floatation of the body

b. Late floatation of the body only in sea water

c. No effect on floatation of body

d. Early floatation of the body

Solution. (c) No effect on floatation of body


Ref: The Essentials of Forensic Medicine & Toxicology, 34 th edition, Dr. K.S. Narayan Reddy,
Page No. 351
Sol:
• The floatation of the body is due to putrefactive gases and is not at all related to the inspired air.
• In India body floats in about 12-18 hours in summer, and 18-36 hours in winter.

Answer. c

94. The test in which weight of lungs is compared with body weight is

a. Ploucquet's Test

b. Fodere's Test

c. Wredin Test

d. Raygat's Test

Solution. (a) Ploucquet's Test


Ref: The Essentials of Forensic Medicine & Toxicology, 34 th edition, Dr. K.S. Narayan Reddy, Page No. 411
Sol:
• As per Fodere’s/Static Test:
Before respiration the weight of the lungs is 30-40g, and after respiration it will be 60-66g. As per Plocquet’s Test, the
ratio between the weight of the lung and body is:1:70 before respiration and 1:35 after respiration.
According to Wredin’s Test, there is replacement of gelatinous tissue in the middle ear by air after respiration.
• Raygat’s test is hydrostatic test.

Answer. a

95. Identify the injury

a. Abrasion

b. Contusion

c. Laceration

d. Incised wound

Solution. (a) Abrasion


Ref: Read the text below
Sol:
• Multiple linear abrasions are seen. This is a graze abrasion.

Answer. a

96. Under which ministry is the Government of India - “Ujalla” scheme?

a. Women & Child Development

b. Petroleum & natural Resources

c. Union Ministry of Power

d. MOHFW

Solution. (c) Union Ministry of Power


The UJALA scheme is implemented by the Energy Efficiency Services Limited (EESL), a joint venture of PSUs under the
Union Ministry of Power.

Answer. c

97. A person developed fever with rash. The child had similar features a week to couple back. No sub clinical cases of the
disease are found. The wife of this person was pregnant. After 8 months they had a daughter but the child had no
symptoms. Mother had taken a vaccine in her adolescent time. Which disease could this be?

a. Rubella

b. Measles

c. Chicken Pox

d. RMSF

Solution. (a) Rubella


Answer. a

98. This diagram was proposed by which committee?

a. Bhore

b. HLEC

c. Krishnan

d. Tendulkar

Solution. (b) HLEC


This is the cube of universal health coverage which was proposed by WHO & in India this is proposed by High Level
Expert committee Group

Answer. b

99. To obtain maximum sample size in observational studies for a given absolute precision prevalence should be:

a. 1

b. 0.5

c. 0.75

d. Any value; depends on power

Solution. (b) 0.5


For observational studies with 95% confidence; sample size formula is n = 4pq/ d 2
Now in this formula q = 1 – p so to get maximum sample size; if p = q = 0.5 or 50%

Answer. b
100. Test of significance applicable here is:

a. Chi-square test

b. Fischer’s exact test

c. Student’s t- test

d. Mann- Whitney test

Solution. (b) Fischer’s exact test


• The diagram is a two by two contingency table where we have only frequency for yes/ no type of data; so non parametric
tests are preferred. Therefore we prefer chi square but in this question in one of the 2 by 2 cells we have value less than 5
so we prefer the fischer exact test.

Answer. b

101. Calculate the Standard error if p is given as 90% and sample size of 100:

a. 1

b. 3

c. 9

d. Can’t calculate with given data

Solution. (b) 3
SE proportion = square root of pq/ n
P = prevalence
For this question we have;
SE proportion = √(90 *10)/ 100 = 3

Answer. b

102. A study was done in different states of India; and it was found that people from Rajasthan have predominantly
vegetarian diet and those from Punjab have non vegetarian diet?

a. Ecological study

b. Case series

c. Case cross over

d. Natural experiment

Solution. (a) Ecological study


The question refers to an ecological study where population has been taken as unit of study

Answer. a

103. Which is not true for?

a. Efficacy: is the effect of an agent under ideal controlled laboratory conditions

b. Effectiveness: is the effect of an agent under real life conditions

c. Efficiency: Measure of relationship between results achieved & effort expended in terms of resources

d. All are true

Solution. (d) All are true


Efficacy: is the effect or usefullness of an agent/ under ideal controlled laboratory conditions
Effectiveness: is the effect or usefullness of an agent/ under real life conditions

Efficiency: Measure of relationship between results achieved & effort expended in terms of money, resources & time

Answer. d

104. In the 2 by 2 table of hypothesis testing. If a pharmacologist developed a drug whose side effect is more than
standard drug but the pharmacologist wants to prove that his drug has similar side effect as standard drug; he will prefer
to do:

a. Power

b. Type II error

c. Type I error

d. Confidence level

Solution. (b) Type II error

Answer. b

105. What change will occur if in a series of numbers of 9,8, 1,8,1,1,4,6,5 an additional 5 is added by mistake?

a. A

b. B

c. C

d. D

Solution. (a) 9,8, 1,8,1,1,4,6,5 an additional 5 is added by mistake


For the original data: 1; 1; 1; 4; 5; 6; 8; 8; 9
• Mode = 1
• Median = 5
• Mean = 43/ 9 = 4.77
Additional 5 will lead to:
• Mode = same
• Median = same
• Mean = 48/ 10 = 4.8 (increase)

Answer. a
106. The 25- 69 years population in the image given below serves as denominator for:

a. Prevalence

b. Incidence

c. Proportional mortality

d. Case fatality

Solution. (b) Incidence


The image shows population at risk which is used as a denominator in case of incidence Prevalence uses total population
as the denominator

Answer. b

107. The diagram will provide a ______ regression coefficient between country and mortality:

a. +1

b. - 1

c. - 0.6

d. + 0.6

Solution. (c) -0.6


The diagram shows relationship between NMR and MMR and socio economic status. This is a negative regression; as
when socio economic status increases then mortality decreases but it won’t be perfectly negative as there are many other
factors which influence the mortality

Answer. c

108. Calculate the sample size for prevalence of 30% with range of 20- 40%. What is n?

a. 21

b. 42

c. 84
d. 400

Solution. (c) 42
• At 95% confidence for observational studies; n = 4pq/ d2
• Now in the question prevalence which is calculated by cross sectional study as per the WHO IEA classification is an
observational study & MC confidence assumed is 95% so it is taken as 95%
• So; for this question n = (4 * 30 * 70)/ (10 * 10) = 84

Answer. c

109. In a study to assess the prevalence of dating among adolescents, schools were selected as boys and girls school as
separate groups randomly. Among the these group of schools the students were selected at random. This type of selection
of students is best described as:

a. Simple random sampling

b. Cluster sampling

c. Stratified sampling

d. Multi-stage sampling

Solution. (c) Stratified sampling


In this question meaningful division has been made that is boys and girls schools have been separately selected for study
of prevalence of dating

Answer. c

110. Which of the following is an advantage of active surveillance?

a. Requires less project staff

b. Is relatively cheaper

c. More accurate

d. Reporting systems can be developed quickly

Solution. (c) More accuruate

Answer. c

111. For a PHC the following data for 1 year is given:


Calculate the CBR:

a. 4.53
b. 20.5

c. 36.2

d. 37.2

Solution. (d) 37.2

Answer. d

112. A 7 day old child at a PHC was diagnosed as jaundice involving palms & soles what is next line of management as
per RMNCHA?

a. Reassure mother

b. Refer to SNCU

c. Refer to NBSU

d. Refer to NBCC

Solution. (c) Refer to NBSU

Answer. c

113. Under PMSMY which colour coding is wrong?

a. Green – Normal pregnancy

b. Red – High risk pregnancy

c. Blue – PIH

d. Orange - Systemic diseases

Solution. (d) Orange - Systemic diseases


• Pradhan Mantri Surakshit Matritwa Yojana (PMSMY)
• On the 9th of every month, free health check ups for pregnant women in 14 states
• If any poor pregnant women comes to private practitioners then, they should give her free treatment
• Launch date: 09th June 2016
• Under MOHFW
• Colour coded:
• Red: High risk Pregnancy
• Blue: PIH
• Yellow: Pregnancy with co morbidities (DM; STIs; Hypothryroid)
• Green: No risk factor

Answer. d

114. Why is Vision 2020 named so?

a. Centres to be developed in multiples of 20’s

b. Objective is to achieve blindness to 0.3% by 2020

c. Incentive for every cataract surgery is Rs. 2020/-


d. Ophthalmologists will be made 2020 per state in the country

Solution. (c) Incentive for every cataract surgery is Rs. 2020/-


Objective of Vision 2020 is to reduce the prevalence of blindness to 0.3%
Vision 2020: The Right to Sight
• Global initiative to reduce avoidable (preventable & curable) blindness by 2020
• Targeted Diseases for Vision 2020:
• World:
• Cataract
• Refractive error & low vision
• Trachoma
• Childhood blindness
• River Blindness

• India: World +
• Glaucoma
• Diabetic Retinopathy
• Corneal blindness
• No river blindness
• Ophthalmologist is present at all levels of health care except Vision where we have a optometrist

Answer. c

115. Which is the following is not correct for prevention of cervical cancer?

a. Pap smear is secondary prevention

b. Cervical cancer vaccine is primary prevention

c. No sexual intercourse form of primordial prevention

d. Pap smear is primary prevention

Solution. (d) Pap smear is primary prevention


Prevention: Ask only two questions to yourself in sequence to get answers for preventive strategies
• Q.1. What it prevents?
• Primordial: Risk factor
• Primary: Disease
• Secondary: Complication
• Tertiary: Further Complication
• Quaternary: Over Diagnosis
Explanation (contd.)
• . At what stage it prevents?
• Pre pathogenesis: Primordial & primary
• Pathogenesis: Secondary & Tertiary
• Strategies:
• Primordial: Health Promotion
• Primary: Health Promotion & Specific Protection
• Secondary: Screening for disease (Screening for risk factor is primary)
Tertiary: Disability Limitation & Rehabilitation

Answer. d

116. Pie in sky is a feature of :

a. Parietal lobe lesion

b. Temporal lobe Lesion


c. Occipital lobe lesion

d. Frontal lobe lesion

Solution. (b) Temporal lobe Lesion


Ref: Read the text below
Sol :
• This is lesion of inferior fibres of optic radiations which loop around the temporal horn , called Meyers loop .

Answer. b

117. Conditions associated with the picture include except:

a. Trisomy 22

b. Trisomy 21

c. Trisomy 18

d. Trisomy 13

Solution. (b) Trisomy 21


Ref: Read the text below
Sol :
• Coloboma of the cornea, iris, ciliary body, choroid, retina and/or optic nerve arise from failed or incomplete closure
of the embryonic fissure during development. This process occurs on day 33 of gestation.
• There are numerous syndromic and chromosomal abnormalities associated with uveal coloboma including
trisomies, duplications, deletions and pericentric inversions. Few are
Trisomy 13 (Patau syndrome)
Trisomy 18 (Edwards syndrome)
13q deletion syndrome
Cat-eye syndrome - chr 22
Aicardi syndrome
MIDAS syndrome
CHARGE syndrome

Answer. b

118. Most common complication of Morgagnian cataract is :

a. Phacotoxic glaucoma

b. Phacomorphic glaucoma

c. Phacoanaphylactic glaucoma

d. Phacolytic glaucoma

Solution. -(d) Phacolytic glaucoma


Ref: Read the text below
Sol :
• Phacolytic glaucoma occurs due to leakage of lens protein (from the hypermature cataract) , into the aqueous ,
blocking the trabecular meshwork.

Answer. d

119. Lacquer cracks are seen in

a. Descements membrane

b. Bowmans membrane
c. Bruchs membrane

d. Basement membrane

Solution. (c) Bruchs membrane


Ref: Read the text below
Sol :
• These are cracks in the Bruchs membrane seen in pathological myopia .

Answer. c

120. Which of the following investigation does not require dilatation of pupil :

a. Retinoscopy

b. Fundoscopy

c. OCT

d. Gonioscopy

Solution. (d) Gonioscopy


Ref: Read the text below
Sol :
• Any procedure at the angle requires a constricted pupil .

Answer. d

121. 15 Critical angle of the cornea is :

a. 26 degrees

b. 36 degrees

c. 46 degrees

d. 56 degrees

Solution. (c) 46 degrees


Ref: Read the text below
Sol :
• Critical angle is the maximum angle of incidence on which refraction is possible.
• Above the critical angle , there is total internal reflection .

Answer. c

122. Comment on the Diagnosis

a. Xanthelasma

b. Lipodermoid

c. Granuloma

d. Squamous papilloma

Solution. (a) Xanthelasma


Ref: Read the text below
Sol :
• Xanthelasma are a type of xanthoma appearing on the eyelids.
• Xanthomas are depositions of yellowish cholesterol-rich material that can appear anywhere in the body in various
disease states.
• They are cutaneous manifestations of lipidosis in which lipids accumulate in foam cells within the skin.
• They are often associated with hyperlipidemias, of both primary and secondary types. Some occur with
altered lipoprotein composition or structure, such as lowered high-density lipoprotein (HDL) levels.
• They frequently occur in patients with type II hyperlipidemia and in the type IV phenotype.They can be seen
without elevated cholesterol also.

Answer. a

123. Most dangerous foreign body is

a. Iron

b. Copper

c. Glass

d. Wood

Solution. (b) Copper


Ref: Read the text below
Sol :
• Pure copper in the eye can cause Pthisis bulbi ( blind shrunk eye )

Answer. b

124. Definition of blindness according to NPCB is BCVA in the better eye of :

< 1/60>a.

< 3/60>b.

< 6/60>c.

d. Finger Count

Solution. (b) < 3/60


Ref: Read the text below
Sol :
• Recently NPCB has changed its definition of blindness and now the criteria is same as WHO .
• Blindness is BCVA of < 3/60 in the better eye or visual field of < 10 degrees in the better eye .

Answer. b

125. This patient complains of foreign body sensation and photophobia.There are no signs of cells in the anterior
chamber or foreign body. Which of the following is true:

a. The patient usually have unilateral red eyes in primary position

b. The patient is likely to be female than male

c. Follicular conjunctivitis of the upper lid is a feature

d. The cornea is not affected

Solution. -(b) The patient is likely to be female than male


Ref: Read the text below
Sol :
Superior limbic keratoconjunctivitis
• It is a rare chronic inflammatory disease of the superior bulbar conjunctiva, limbus and upper cornea of unknown
etiology.
• This disease has been associated with thyroid dysfunction, keratoconjunctivitis sicca and rheumatoid arthritis.
• It is characterized by marked inflammation of the upper tarsal and bulbar conjunctiva, fluorescein staining of the cornea
and upper limbus.
• The age of presentation is around the sixth decade of life, affecting women more often than men (ratio 3:1)

Answer. b

126. A patient with history of smoking presents with a small <1 cm endobronchial nodule which was biopsied and studied.
Electron microscopy showed the following. Which marker on immunohistochemistry would help reach a diagnosis?

a. TTF1

b. Napsin A

c. Synaptophysin

d. Calretinin

Solution. C
The location, size and electron microscopy picture of this lesion goes in favour of a carcinoid tumor or a neuroendocrine
tumor. The best marker to identify such lesions is synaptophysin and chromogranin.

Answer. c

127. A 58 year old woman with breast cancer is being evaluated for further management. On IHC the following findings
were obtained. Which of the following is true about the course of this patient’s condition?

a. Trastuzumab is a treatment option

b. Hormonal therapy is not a treatment option

c. Patient has favourable prognosis

d. Patient falls into Basal subgroup of breast cancers

Solution. C
The IHC pictures show nuclear positivity for ER and no membranous positivity for Her 2 Neu. This falls under the Luminal
A group of molecular classification and has the best prognosis among all the subtypes. Since Her 2 Neu is negative,
Trastuzumab is not an option for treatment but since ER is positive, hormonal therapy can be used. Basal subtype is triple
negative, having one of the worst prognosis of all the tumors.

Answer. c

128. An 8 year old male patient with history of hematuria 3 days after contracting a respiratory infection is being
evaluated. Which of the following is most likely to be the electron microscopy finding?
a. Basket weave pattern of the basement membrane

b. Subepithelial deposits

c. Mesangial deposits

d. Subendothelial deposits

Solution. C
Since the history is short, the possibility of acquiring IgA nephropathy is higher. It is characterised by mesangial deposits.

Answer. c

129. Krukenbergtumor of the ovary can arise from all of the following primary sites except

a. Stomach

b. Appendix

c. Breast

d. Liver

Solution. D
Krukenbergtumor usually is characterised by mucinous or signet ring cell morphology, which can be seen in all of the
options commonly except liver. Hence, the best possible answer is liver.

Answer. d

130. Which of the following genetic changes in medulloblastoma indicates the best prognosis?

a. Sonic hedgehog pathway

b. Wnt signalling pathway

c. Isochromosome 17q

d. Myc amplifications

Solution. B
Medulloblastoma is divided into 4 molecular subtypes- Wnt signalling pathway, Sonic Hedgehog Pathway, Group 3 and
Group 4. Wnt signalling pathway has the best prognosis while group 3 carries the worst.

Answer. b

131. Risk of ventricular rupture is maximum at what duration post myocardial infarction

a. 1-4 days

b. 5-9 days

c. 2 weeks

d. 4 weeks

Solution. A
Answer. a

132. All of the following are characteristic features of malignancy except

a. Atypical mitosis

b. Hyperchromatic nuclei

c. Loss of polarity

d. Chromosomal diploidy

Solution. D
Chromosomal aneuploidy is seen in malignancies

Answer. d

133. All of the following features can help differentiate Crohn’s disease from Ulcerative colitis except

a. Granuloma formation

b. Crypt abscesses

c. Transmural inflammation

d. Pseudopoylps

Solution. B
Cryptitis and crypt abscesses maybe be seen in both. Granuloma formation and transmural inflammation are
characteristic of CD while pseudopolyposis is characteristic of UC.

Answer. b

134. Lardaceous spleen occurs due to

a. Deposition of amyloid in white pulp

b. Deposition of amyloid in red pulp

c. Deposition of hemosiderin

d. Deposition of calcium

Solution. B
Red pulp involving sinuses are the site of amyloid deposition in lardaceous spleen

Answer. b
135. Tumor marker(s) associated with both colon cancer and pancreatic cancer is

a. CEA

b. CA-125

c. CA 15-3

d. All of the above

Solution. A
CEA or carcinoembryonic antigen is seen in both colon and pancreatic cancers and other adenocarcinomas as well. CA
125 is a marker for ovarian carcinoma and CA 15-3 is a marker for breast carcinoma.

Answer. a

136. Which of the following has the least chance of metastasising to the brain?

a. Lung carcinoma

b. Melanoma

c. Breast carcinoma

d. Prostate carcinoma

Solution. D
Most common primary tumors metastasising to brain are lung, breast, melanoma, kidney and GIT. Prostate carcinoma has
a tendency to primarily involve the bones, not brain.

Answer. d

137. Bone marrow of a patient suffering from immune thrombocytopenic purpura will show

a. Absent megakaryocytes

b. Megakaryocyte hyperplasia

c. Micromegakaryocytes

d. Fragmented megakaryocytes

Solution. B
ITP occurs due to peripheral destruction of platelets and the marrow responds by increasing the production of
megakaryocytes.

Answer. b

138. All are true about paroxysmal nocturnal hemoglobinuria except

a. Acquired genetic defect

b. Thrombosis is a common complication

c. May progress to AML

d. Hematopoetic stem cell transplant will not be useful

Solution. D
Since the acquired genetic defect is in the stem cell, a transplant is potentially curable. Around 5-10% cases of PNH can
progress to AML or MDS.

Answer. d

139. Faggot cells are seen in

a. Whipple’s disease

b. Hodgkin lymphoma

c. Oligoblasticleukemia
d. AML with PML RARA fusion

Solution. D
Faggot cells are atypical promyelocytes bearing stacks of Auer rods.

Answer. d

140. Coffee bean nuclei are seen in all except

a. Brenner tumor

b. Langerhan’s cell histiocytosis

c. Follicular carcinoma thyroid

d. Chondroblastoma

Solution. C
A longitudinal groove in the nucleus is seen in tumor cells of Brenner tumor, Granulosa cell tumor, LCH, Chondroblastoma
and papillary carcinoma thyroid.

Answer. c

141. A duodenal biopsy from an HIV positive patient with severe abdominal pain, vomiting and diarrhoea shows the
following. What is your diagnosis?

a. Cryptosporidium

b. Giardia

c. Strongyloides

d. Trichura

Solution. C
Strongyloides usually sits in the crypts of duodenum and with increasing severity invades deeper layers.

Answer. c

142. All of the following are true about the following tumor except
a. Can express progesterone receptor

b. Rhabdoid morphology is grade III

c. Bone invasion upgrades the tumor

d. EMA positivity is seen

Solution. C
Bone invasion does not upgrade meningioma. Chordoid and clear cell morphology is grade II and rhabdoid and papillary
morphology is grade III. EMA and PR are expressed.

Answer. c

143. Warburg effect in neoplastic cells is classically associated with which of the following genes?

a. MYC

b. KRAS

c. Rb

d. PAX 8

Solution. A
Pro-growth pathways upregulateexpression of the transcription factor MYC, which drives changes in gene expression that
support anabolic metabolism and cell growth. Among the most important metabolic factors that are upregulated by MYC
are multiple glycolytic enzymes and glutaminase, which is required for mitochondrial utilization of glutamine.

Answer. a

144. Satellite cells are seen in

a. Muscle

b. Liver

c. Skin

d. Eye

Solution. A
Satellite cells are adult stem cells located in the muscle.

Answer. a

145. Most common soft tissue sarcoma in adults is

a. Liposarcoma

b. Undifferentiated pleomorphic sarcoma

c. Leiomyosarcoma

d. Synovial sarcoma
Solution. B
UPS or Malignant Fibrous Histiocytoma is the most common soft tissue sarcoma in adults. Most common retroperitoneal
soft tissue sarcoma is liposarcoma.

Answer. b

146. Just discharged from hospital after a syncope of unknown origin,Now a patient is complaining of twitching
and malaise with no angina,Comment on the given ECG

a. Atrial fibrillation

b. Anterior STEMI

c. Posterior STEMI

d. Inferior stemi

Solution. (c) Posterior STEMI


Ref: Read the text below
Sol:
As the posterior myocardium is not directly visualised by the standard 12-lead ECG, reciprocal changes of STEMI are
sought in the anteroseptal leads V1-3. Posterior MI is suggested by the following changes in V1-3:
• Horizontal ST depression
• Tall, broad R waves (>30ms)
• Upright T waves
• Dominant R wave (R/S ratio > 1) in V2

Answer. c

147. Which of the following is a late electrocardiogram finding in patients with pericarditis?

a. "Knuckle sign" in aVR.

b. PR depression.

c. T-wave inversions.

d. ST elevation.

Solution. (c) T-wave inversions.

Ref: Read the text below

Sol:

·      Initially (Stage 1), patients have the classic findings of diffuse ST elevation, PR depression, and a "knuckle
sign" in the PR segment of aVR.

·      In Stage 2, the ST segment returns to baseline and the T-wave amplitude decreases. T-wave inversions are
classically a late finding (Stage 3) in the electrocardiograms of patients with pericarditis.

·      Stage 4 is the resolution of the repolarization abnormalities.

Answer. c

148. Regarding “N-95 Respirator Mask” :


a. Filter efficiency of 90 % or more against particulate aerosols is required.

b. Filter efficiency of 95 % or more against particulate aerosols is required.

c. Filter efficiency of 99 % or more against particulate aerosols is required.

d. Filter efficiency of 65 % or more against particulate aerosols is required.

Solution. b) Filter efficiency of 95 % or more against particulate aerosols is required.


Ref– Read the text below
Sol:

Answer. b

149. Which of the following is not associated with cardiac tamponade?

a. Pulsus paradoxus.

b. Narrow pulse pressure.

c. Electrical alternans.

d. Left ventricular hypertrophy.

Solution. (d) Left ventricular hypertrophy.


Ref: Read the text below
Sol:
• Patients with cardiac tamponade usually do not have underlying heart disease. The most common causes are
malignancy and pericarditis.
• In fact, the electrocardiogram classically has very low voltage.
• Electrical alternans (beat-to-beat amplitude variation) is seen in about 20% of cases. Pulsus paradoxus and a narrow
pulse pressure are commonly seen in tamponade.

Answer. d

150. All the following criteria support the diagnosis of Kawasaki's disease, except:

a. Platelet count >1000 x 103/μL

b. Fever for 5 days

c. Cervical lymphadenopathy

d. Erythema and edema of the hands or feet

Solution. (a) Platelet count >1000 x 103/μL

Ref– Read the text below


Sol:
• Kawasaki's disease is an acute vasculitis of unknown origin.
• Diagnostic criteria are fever for 5 days without another source and 4 of the following features: (1) rash, (2) nonexudative
conjunctivitis, (3) mucous membrane involvement, (4) edema and/or erythema of the hands or feet, (5) cervical
adenopathy of 1.5 cm or more.
• A platelet count of more than 1000 x 103/μL is frequently seen and may aid in diagnosis but is not a diagnostic
criteria.

Answer. a
151. An infarct involving the seventh nerve and nucleus is likely to be secondary to occlusion in the territory of the :

a. The Basilar Artery

b. The superior cerebellar artery

c. The anterior inferior cerebellar artery

d. The posterior inferior cerebellar artery

Solution. (c) The anterior inferior cerebellar artery


Ref– Read the text below
Sol:
• The AICA territory includes the middle cerebellar peduncle, fifth nerve sensory nucleus and tract, seventh nerve
and nucleus, eighth nerve, vestibular nuclei and the descending sympathetic tracts.
• AICA-related lateral Pontine infarction produces a syndrome similar to Wallenberg's; but instead of hoarseness,
palatal weakness and loss of taste, there is ipsilateral facial paralysis and deafness.

Answer. c

152. Identify the type of breathing shown in the picture:

a. Apnuestic breathing

b. Ataxic breathing

c. CheyneStoke’s breathing

d. Biot’s breathing

Solution. (b) Ataxic breathing


Ref: Read the text below

Sol:
Ataxic breathing
- characterized by complete irregularity of breathing, with irregular pauses and increasing periods of apnea.
- As the breathing pattern deteriorates, it merges with agonal respirations.
- It is caused by damage to the medulla oblongata due to strokes or trauma

Answer. b

153. The most likely diagnosis in the case of a patient with multiple pulmonary cavities, haematuria and red cell
casts is :

a. Anti – GBM disease

b. Churg-Strauss allergic granulomatosis

c. Systemic lupus erythematosus

d. Wegner’s granulomatosis

Solution. (d) Wegner’s granulomatosis


Ref: Nelson - 731.
Sol :

Answer. d
154. N-acetyl penicillamine is used in the treatment of poisoning by :

a. Mercury

b. Lead

c. Cadmium

d. Arsenic

Solution. (a) Mercury


Ref.: Read the text below
Sol :
Penicillamine :
- Drug of choice for Cu poisoning.
- Alternative drug to dimercaptrol for Hg poisoning.
- May be used as an adjuvant to CaNa2 EDTA in chronic lead poisoning.

Answer. a

155. HLA B27 is usually detected in the following conditions except :

a. Systemic lupus erythematosus

b. Ankylosing spondylitis

c. Psoriatic arthritis

d. Reactive arthritis

Solution. (a) Systemic lupus erythematosus


Ref.: Harrison’s - 2026
Sol :
HLA B27
• Ankylosing spondylitis
• Reiter’s symdrome
• Reactive arthritis (Yersinia, Salmonella, Gonococcus)
• Psoriatic arthritis
• Acute anterior uveitis
• Juvenile rheumatoid arthritis.

Answer. a

156. Which of these medications is concentrated in adrenergic nerve terminals by the amine uptake pump?

a. Hemicholinium-3

b. Vesamicol

c. Alphamethyl-DOPA

d. Guanethidine

Solution. (d) Guanethidine


Ref– Read the text below
Sol:
• Inhibitors of catecholamine release act by preventing the release of catecholamine from the nerve terminal
following a nerve action potential.
• Such agents include bretylium (Bretylol) and guanethidine (Ismelin). Like reserpine, guanethidine causes
depletion of catecholamines. Thus, guanethidine causes both inhibition of the release of catecholamines and depletion
of catecholamines.
• Guanethidine is concentrated in adrenergic nerve terminals by the amine uptake pump.
• Therefore, agents which inhibit the amine uptake pump, such as imipramine, can antagonize the effects of
guanethidine.

Answer. d

157. Which of the following is the drug of choice for hypertensive encephalopathy?

a. Sodium nitroprusside.

b. Nifedipine.
c. Hydralazine.

d. Methyldopa.

Solution. (a) Sodium nitroprusside.


Ref: Read the text below
Sol:
• Sodium nitroprusside is the drug of choice for hypertensive encephalopathy. Patients require immediate and
titratable lowering of the blood pressure.
• Sodium nitroprusside has a rapid onset with a relatively short half-life. Intravenous nitroglycerin and labetalol have
been used, but nitroprusside is still the best agent.
• The other choices are not well suited for the emergency setting because they are difficult to adjust quickly if the patient
becomes too hypotensive.
• The patient's mean arterial pressure should not be lowered by more than 20-25% within the first hour.

Answer. a

158. What clinical finding is most commonly associated with the onset of idiopathic thrombocytopenic purpura
(ITP)?

a. Ecchymoses.

b. Purpura.

c. Petechiae.

d. Gingival bleeding.

Solution. (c) Petechiae.


Ref: Read the text below
Sol:
• Petechiae are most commonly associated with the onset of ITP.
• Ecchymoses, purpura, gingival bleeding, epistaxis, and menorrhagia may also develop.
Otherwise, the physical examination will be normal.

Answer. c

159. Recommended sequential treatment for migraine is:

a. Dihydroergotamine (DHE), then metoclopramide.

b. Sumatriptan, then dexamethasone.

c. Metoclopramide, then meperidine.

d. DHE, then sumatriptan.

Solution. (b) Sumatriptan, then dexamethasone..


Ref: Read the text below
Sol:
• Dexamethasone is effective in reducing the rate of recurrent migraine following standard treatment.
• Pretreatment with an antiemetic prior to DHE use is recommended due to the high incidence of vomiting with DHE.
• Meperidine is less effective than other agents, and the frequent use of opioids for recurrent headaches may lead to
adverse effects or exacerbate headaches.
• Sumatriptan should not be given within 24 hours of DHE or other ergot use because both cause
vasoconstriction.

Answer. b

160. Which of the following is not a glycoprotein IIb/IIIa inhibitor:

a. Abciximab

b. Clopidogrel

c. Eptifibatide

d. Tirofiban

Solution. (b) Clopidogrel


Ref: Read the text below
Sol:
• Glycoprotein IIb/IIIa inhibitors inhibit the final common pathway of platelet aggregation. Clopidogrel
inhibits ADP dependant platelet aggregation
Answer. b

161. Most common symptom of brain abscess is?

a. Vomiting.

b. Fever.

c. Confusion.

d. Headache.

Solution. (d) Headache.


Ref: Read the text below
Sol:
• Presenting features of brain abscess are nonspecific, but headache is present in almost all cases. Meningeal signs like
neck stiffness are uncommon.
• Focal neurologic signs are seen in about 33-60% of cases, and symptoms of increased intracranial pressure such as
vomiting, confusion, and obtundation are seen about half of the time.

Answer. d

162. Which is the wave in the EEG tracing pointed by the arrow in the figure:

a. Alpha

b. Beta

c. Theta

d. Delta

Solution. (c) Theta


Ref: Read the text below
Sol:
- Theta wave has its origin from the hippocampus
- Frequency: 4- 7 Hz
Answer. c

163. Infection most commonly associated with acute uncomplicated bronchitis is:

a. Influenza.

b. Bordetella pertussis.

c. Chlamydia.

d. Streptococcus pneumoniae.

Solution. :(a) Influenza.


Ref: Read the text below
Sol:
• The respiratory viruses, influenza, parainfluenza, and respiratory syncynctial virus, are the agents most
commonly associated with acute uncomplicated bronchitis.
• B. pertussis and C. pneumoniae are also associated with bronchitis but generally present with a more prolonged or
persistent course of cough.
• S. pneumoniae is associated with community-acquired pneumonia as well as being part of normal oral flora, and its
impact on the overall incidence of acute bronchitis is unclear.

Answer. a

164. Most common site of infection in a renal transplant patient during the first year posttransplant is:

a. Urinary tract.

b. Mucocutaneous tissue.

c. Respiratory tract.

d. Cerebrospinal fluid.

Solution. :(b) Mucocutaneous tissue.


Ref: Read the text below
Sol:
• Renal transplant patients receive aggressive immunosuppression therapy until 3-6 months post-transplant.
• In the first year after transplant, 40-80% of patients will experience at least one infection. The most common
infections are mucocutaneous, commonly oropharyngeal, esophageal, or vaginal candidiasis.
• Urinary tract infections and respiratory infections are the next most likely sites of infection in the first
posttransplant year, with meningitis being less likely.

Answer. b

165. A chronic renal dialysis patient is brought to the emergency in cardiac arrest. Most likely cause is:

a. Pericardial effusion.
b. Hyperkalemia.

c. Malignant hypertension.

d. Postdialysis hypotension.

Solution. (b) Hyperkalemia..


Ref: Read the text below
Sol:
• Although all of the choices can lead to cardiac arrest in chronic dialysis patients,
hyperkalemia is the most common cause.
• Treatment should start with intravenous calcium, then dextrose and insulin, as well as sodium bicarbonate.
• Other electrolyte disturbances seen in uremic patients include hypokalemia, hypocalcemia, and
hypermagnesemia.

Answer. b

166. Which of the following signs or symptoms is consistent with severe asthma attack.?

a. Silent chest, peak expiratory flow rate (PEFR) < 33% of predicted.

b. Tachycardia >110/minute, PEFR < 50% of predicted.

c. Exhaustion, hypotension.

d. PCO2 normal or high on arterial sampling.

Solution. (b) Tachycardia >110/minute, PEFR < 50% of predicted..


Ref: Read the text below
Sol:
Acute exacerbations of asthma can be defined as severe or life-threatening. Criteria for diagnosis are as
follows:
Severe attack
• Unable to complete sentences
• Respiratory rate of greater than 25/minute
• Heart rate greater than 110 beats per minute
• PEFR of less than 50% of predicted
Life-threatening
• Cyanosis, very poor respiratory effort

• Silent chest
• Bradycardia and hypotension
• PEFR <33% predicted
Arterial blood sampling will show normal or high PCO2 and PO2 less than 8kPa. In any case of asthma
exacerbation a chest X-ray should be ordered to exclude a pneumothorax and to look for a concurrent
pneumonia. The principles for management of exacerbations includes
• Oxygen
• Salbutamol
• Hydrocortisone (or prednisolone)
• Inhaled steroids
• Theophylline (be aware of risks of toxicity – if not already on regular theophyllines will need a loading dose)

Answer. b

167. An electrocardiogram change consistent with myxedema coma is:

a. Prolongation of the QT interval.

b. Premature ventricular contractions.

c. Premature atrial contractions.

d. Left ventricular hypertrophy.

Solution. :(a) Prolongation of the QT interval.


Ref: Read the text below
Sol:
• Electrocardiogram changes seen in myxedema coma include sinus bradycardia,
prolongation of the QT interval, and low voltage with flattening or inversion of T waves.

Answer. a

168. The virus that has been associated with progressive multifocal leukoencephalopathy is
a. SV 40

b. Polyomavirus

c. BK virus

d. JC virus

Solution. (d) JC virus


Ref: Read the text below
Sol:
• Progressive multifocal leukoencephalopathy is an uncommon, generally fatal,
demyelinating disease that occurs in compromised hosts.
• It is caused by infection of oligodendrocytes by JC virus.
• There is no known treatment for this viral disease.

Answer. d

169. The most common infectious cause worldwide of primary adrenal insufficiency is ?

a. Human immunodeficiency virus.

b. Cytomegalovirus.

c. Tuberculosis.

d. Pseudomonas organisms.

Solution. :(c) Tuberculosis.


Ref: Read the text below
Sol:
• Worldwide, tuberculosis is the most common infectious cause of primary adrenal insufficiency.
• In the developed countries, however, infection with the human immunodeficiency virus (HIV) is the most
common cause.
• HIV may cause adrenal insufficiency via opportunistic infections (principally cytomegalovirus), the use of medications
such as ketokonazole, or inhibition of the hypothalamicpituitary- adrenal (HPA) axis by cytokines released by
macrophages

Answer. c

170. What is the area marked ‘X’

a. ST segment

b. ST interval

c. SU segment

d. QT interval

Solution. (b) ST interval


Ref: Read the text below
Sol:
Answer. b

171. Which of the following agent inactivates the infectious agent associated with Creutzfeldt-Jakob disease

a. Catalases

b. Hyaluronidases

c. Nucleases

d. Proteases

Solution. (d) Proteases


Ref: Read the text below
Sol:
• Prions are infectious particles, which can transmit a disease.
• These are composed chiefly a protein without any detectable nucleic acid.The infectious agent of
Creutzfeldt-Jakob disease is a prion which is inactivated by proteases.

Answer. d

172. The minimal concentrate on of alcohol necessary to kill bacteria and enveloped viruses is

a. 40%

b. 50%

c. 60%

d. 70%

Solution. (d) 70%


Ref: Read the text below
Sol:
• Alcohol acts by protein coagulation.
• An alcohol concentration of 70% to 95% is necessary to kill bacteria.

Answer. d

173. A colonic cancer individual developed pneumonia. Sputum culture for fungus shown growth. LPCB shown in
the picture. What is the causative agent?
a. Candida tropicalis

b. Aspergillus fumigatus

c. Cryptococcus neoformans

d. Penicillium marneffi

Solution. (b) Aspergillus fumigatus


Ref: Read the text below
Sol:
• LPCB reveals Aspergillus fumigatus
• A. fumigatus is a ubiquitous, filamentous fungus whose airborne spores are constantly in the air.
• Aspergilli have characteristic septate hyphae branching dichotomously at acute angles.
• Allergic bronchopulmonary aspergillosis is an allergic disease in which the organism colonizes the mucous plugs formed
in the lungs but does not invade lung tissues. It is diagnosed by the finding of high titers of IgE.
• Aspergilloma (fungus ball) is a roughly spherical growth of Aspergillus in a preexisting lung cavity; growth does not
invade the lung tissues. It presents clinically as recurrent hemoptysis and is diagnosed by radiologic methods; an “air
sign” shift will be seen with a change in thep osition of the patient.
• Invasive aspergillosis is most common in patients with severe neutropenia starting in the lungs or spreading from
sinus colonization. It requires aggressive treatment with voriconazole or lipid formulation of amphotericin B.

Answer. b

174. One week after delivery, an infant boy becomes extremely irritable and continuously rubs his left eye, which
contains a mucopurulent exudate. An examination of the eye shows papillae on the conjunctiva. No bacteria are seen
in the Gram stain. The most likely causative infectious agent is

a. Candida albicans

b. Chlamydia trachomatis

c. Cytomegalovirus

d. Herpes simplex virus

Solution. (b) Chlamydia trachomatis


Ref: Read the text below
Sol:
• This case of neonatal conjunctivitis is most likely caused by Chlamydia trachomatis since no bacteria are present
and papillae rather than follicles are observed.
• The mother was probably the source of the infection.

Answer. b

175. A woman from china undergoes a spontaneous abortion, and in the recovery room she passes numerous
proglottids. The medical technologist injects one of the proglottids with India ink to delineate the uterus (shown in the
image). How did she become infected with this tapeworm?
a. Person-to-person or direct fecal-oral exposure

b. Consumption of insufficiently cooked sirloin

c. Consumption of insufficiently cooked pork sausage

d. Consumption of sushi

Solution. (b) Consumption of insufficiently cooked sirloin


Ref: Read the text below
Sol:
• Based on the number of uterine branches stained by the India ink, the correct identification is Taenia saginata or
commonly referred to as the beef tapeworm.
• The bladder worm or Cysticercus bovis is the infective stage for people, and the vehicle for transmission is
insufficiently cooked beef.
• The parasite may only produce vague abdominal discomfort, and the patient becomes aware of an infection when
proglottids are noticed in the undergarments or after a bowel movement.

Answer. b

176. A 67-year-old woman is hospitalized with a fever of unknown origin. An elevated C-reactive protein (CRP) and
erythrocyte sedimentation rate (ESR) suggest an ongoing systemic inflammatory response.Which cytokine is
especially important in inducing elevations in CRP and ESR?

a. Interleukin-4

b. Interleukin-10

c. Interleukin- 6

d. Interleukin-12

Solution. (c) Interleukin- 6


Ref: Read the text below
Sol:
• When the body experiences a significant inflammatory reaction, many physiological responses are generated.
Interleukin-6, released into the circulation from activated macrophages and other cells, stimulates hepatocytes to
release acute phase reactants.

• These proteins include components of the complement system, clotting system, and variety of others that play a role in
promoting inflammation, or serving as opsonins.

Answer. c

177. An 18-year-old woman preparing to enter college is required to be up-to-date on vaccinations. In addition to
required childhood vaccinations, students entering college are often required to be vaccinated against which
organism?

a. Chlamydia trachomatis

b. Hepatitis C virus

c. Human papillomavirus

d. Neisseria meningitidis
Solution. (d) Neisseria meningitidis
Ref: Read the text below
Sol:
• Infection with this agent is most common in children; however, young adults in crowded conditions such as
college dormitories are also at high risk.
• Many colleges and universities require or strongly recommend vaccination for their incoming students.
• The only other agent listed for which a vaccine is available is human papillomavirus.

Answer. d

178. Which of the following components are found in the cell wall of gram-positive bacteria, but not gram-negative
bacteria.

a. Cytoplasmic membrane

b. Lipopolysaccharide

c. Outer membrane

d. Teichoic acid

Solution. (d) Teichoic acid


Ref: Read the text below
Sol:
• Techoic acids are found in the cell envelope of only Gram Positive bacteria.

Answer. d

179. Process by which bacterial or plasmid DNA may be mistakenly incorporated into one phage being produced
by the lytic life cycle and then that DNA-transferred to another bacterial cell which may acquire some new genetic
traits is called

a. Conjugation

b. Generalized transduction

c. Homologous recombination

d. Site-specific recombination

Solution. (b) Generalized transduction


Ref: Read the text below
Sol:
• Generalized transduction is a process of horizontal gene transfer wherein bacterial genes are transferred from one
to the other bacterium by a lytic/virulent phage.

Answer. b

180. A 35-year-old male patient presents with numerous subcutaneous hemorrhages. History and physical
examination reveal that he has been taking sedormid (a sedative) for the past week. Laboratory tests indicate
normal hemoglobin and white blood cell levels with significant thrombocytopenia. You suspect that he has
developed a drug induced type II hypersensitivity reaction. This reaction may occur if the drug does which of the
following?

a. Activates cytotoxic cells

b. Acts as a hapten

c. Induces oxygen radical production through the respiratory burst pathway

d. Persists in macrophages

Solution. (b) Acts as a hapten


Ref: Read the text below
Sol:
• Hapten is an incomplete antigen which is associated with antigenicity but no immunogenicity.
• Haptens can be converted into complete antigens by combining with a protein carrier molecule.

Answer. b

181. Each of the following statements concerning D. latum are correct except:
a. It is transmitted by undercooked fish

b. It has operculated eggs

c. Crustaceans are intermediate host for D. latum

d. It has scolex with a circle of hooks

Solution. (d) It has scolex with a circle of hooks


Ref: Read the text below
Sol:
• Fish tape worm scolex does not contain circle of hooks and hence referred as unarmed.

Answer. d

182. All of the following picornaviruses are resistant to the acidity of stomach except

a. Coxsackie’s virus A

b. Echoviruses

c. Coxsackie’s virus B

d. Rhinovirus

Solution. (d) Rhinovirus


Ref: Read the text below
Sol:
• Rhinoviruses are an exception among picronaviruses as they are acid labile.

Answer. d

183. A divorced working mother takes her 4 year old child to day care center. She has noticed that the child’s
frequent stools are non-bloody, foul smelling and fatty. The child has no fever. One should suspect infection with
which parasite?

a. Amebiasis

b. Ascariasis

c. Enterobiasis

d. Giardiasis

Solution. (d) Giardiasis


Ref: Read the text below
Sol:
• Giardiasis is associated with malaabsorption syndrome often presenting as steatorrhea.

Answer. d

184. A single, 30 year old woman presented to her physician with vaginitis. She complained of a slightly increased,
malodorous discharge that was grey-white in color, thin, and homogenous. Clue cells were discovered when the
discharge was examined microscopically. Which organism listed below was the most likely cause of her infection?

a. Candida albicans

b. Trichomans vaginalis

c. Escherichia coli

d. Mobiluncus spp.

Solution. (d) Mobiluncus spp.


Ref: Read the text below
Sol:
• This the clinical history of bacterial vaginosis in which protective bacterium that is lactobacillus is decreased while
pathogenic bacteria like gardnerella vaginalis, mobiluncus and bacteroides are increased.

Answer. d

185. Chromoblastomycosis is caused by all except


a. Fonsacea Pedrosoi

b. R. aquaspersa

c. F. Compacta

d. E. Jeanselmai

Solution. (d) E. Jeanselmai


Ref: Read the text below
Sol:
• Chromoblastomycosis is a subcutaneous fungal infection caused by Fonsaesea pedrosoi, Fonsaesea
compacta,Cladosporium carionii,Phialophora verrucosa and Rhinocladiella acquaspersa.

Answer. d

186. Which one of the following viruses would be most likely to establish a latent infection?

a. Adenovirus

b. Measles virus

c. Influenza virus

d. Parvovirus

Solution. (a) Adenovirus.


Reference – Read the text below
Sol:
• While the herpesviruses (HSV, CMV, VZV) are all well known for latency, adenovirus can also form a latent infection in
the lymphoid tissue.
• In 50 to 80% of surgically removed tonsils or adenoids, adenovirus can be cultured. The virus has also been
cultured from mesenteric lymph nodes, and, in rare cases, viral DNA has been detected in peripheral lymphocytes.
• Recurrent illness usually does not arise from these latent infections; however, activation can occur in the
immunosuppressed.

Answer. a

187. Chicken pox is a common disease of childhood. It is caused by which of the following viruses?

a. Cytomegalovirus

b. Rotavirus

c. Varicella-zoster virus

d. Adenovirus

Solution. (c) Varicella-zoster virus.


Reference – Read the text below
Sol:
• Varicella-zoster virus is a herpesvirus. Chicken pox is a highly contagious disease of childhood that occurs in the late
winter and early spring. It is characterized by a generalized vesicular eruption with relatively insignificant systemic
manifestations.
• Adenovirus has been associated with adult respiratory disease among newly enlisted military troops.
Crowded conditions and strenuous exercise may account for the severe infections seen in this otherwise
healthy group.
• Papillomavirus is one of two members of the family Papovaviridae, which includes viruses that produce human warts.
These viruses are host-specific and produce benign epithelial tumors that vary in location and clinical appearance. The
warts usually occur in children and young adults and are limited to the skin and mucous membranes.

Answer. c

188. A child has mononucleosis-like symptoms yet the test for mononucleosis and the EBV titers are negative.
Which of the following is one cause of heterophile-negative mononucleosis?

a. Cytomegalovirus

b. Herpes simplex virus

c. Varicella-zoster virus

d. Adenovirus
Solution. (a) Cytomegalovirus.
Reference – Read the text below
Sol:
• Infectious mononucleosis caused by cytomegalovirus (CMV) is clinically difficult to distinguish from that caused by
Epstein-Barr virus. Lymphocytosis is usually present with an abundance of atypical lymphocytes.
• CMV-induced mononucleosis should be considered in any case of mononucleosis that is heterophil-negative and in
patients with fever of unknown origin.

Answer. a

189. Which of the following is the leading cause of bronchiolitis and community-acquired pneumonia in infants?

a. Measles virus

b. Influenza virus

c. Respiratory syncytial virus

d. Parainfluenza virus

Solution. (c) Respiratory syncytial virus.


Reference – Read the text below
Sol:
• Orthomyxoviruses and paramyxoviruses are RNA viruses that contain a single-stranded RNA genome. The influenza
viruses belong to the orthomyxoviruses.
• They cause acute respiratory tract infections that usually occur in epidemics. Isolated strains of influenza virus are
named after the virus type (influenza A, B, or C) as well as the host and location of initial isolation, the year of isolation,
and the antigenic designation of the hemagglutinin and neuraminidase.
• Both the hemagglutinin and neuraminidase are glycoproteins under separate genetic control, and because of
this they can and do vary independently. The changes in these antigens are responsible for the antigenic drift
characteristic of these viruses.
• The paramyxoviruses include several important human pathogens (mumps virus, measles virus, respiratory syncytial
virus, and parainfluenza virus). Both paramyxoviruses and orthomyxoviruses possess an RNA-dependent RNA polymerase
that is a structural component of the virion and produces the initial RNA.
• Respiratory syncytial viruses (RSV) are not related to the paramyxoviruses. They are 150-nm single-stranded RNA
viruses. There are 2 antigen groups, A and B, which play no role in diagnosis and treatment.

Answer. c

190. Tinea capitis is caused by which of the following?

a. E. floccosum

b. M. furfur

c. M. canis

d. E. werneckii

Solution. (c) M. canis.


Reference – Read the text below
Sol:
• Dermatomycoses are cutaneous mycoses caused by three genera of fungi: Microsporum, Trichophyton, and
Epidermophyton. These infections are called tinea or ringworm, a misnomer that has persisted from the days when they
were thought to be caused by worms or lice.
• Tinea capitis (ringworm of the scalp) is due to an infection with M. canis or T. tonsurans. It usually occurs during
childhood and heals spontaneously at puberty. Circular areas on the scalp with broken or no hair are characteristic of this
disorder.
• Tinea corporis (ringworm of the body) is caused by M. canis and T. mentagrophytes. This disorder affects
smooth skin and produces circular pruritic areas of redness and scaling.
• Both tinea cruris (ringworm of the groin, "jock itch") and tinea pedis (ringworm of the feet, athlete's foot) are
caused by T. rubrum, T. mentagrophytes, or E. floccosum. These common conditions are pruritic and can cause scaling.

Answer. c

191. Pt presents with loss of sensation over lateral three and half fingers.

Which will be seen additionally :

a. Loss of sensation over thenar eminence

b. Loss of sensation over hypo thenar eminence

c. Atrophy of adductor pollicis


d. Opponens palsy

Solution. (d)
Explanation :
Median Nerve enters in hand and gives branches superficial and deep to carpal tunnel. Deep Branch further divides into -
• Recurrent branch of Median Nerve – Innervates the thenar muscles ( Opponens Pollicis / Flexor Pollicis brevis Sup
Head / Abductor Pollicis brevis )
• Palmar digital branch – Innervates the palmar surface and fingertips of the lateral three and half digits. Also
innervates the lateral two lumbrical muscles. Superficial Branch supplies skin over the thenar eminence.

Answer. d

192. Bone and Joint Day is celebrated on :

a. 3rd June

b. 4th August

c. 12th October

d. 16th October

Solution. (b)
Explanation :
CTEV Day : 3rd June
Bone and Joint Day : 4th August
Spine Day : 16th October
Arthritis Day : 12th October

Answer. b

193. First clinical stage of Fracture union corresponds to :

a. Callus Formation

b. Woven Bone Formation

c. Lamellar Bone Formation

d. Hematoma Formation

Solution. (b)
Explanation :
First Clinical stage of fracture union : Woven Bone
First Radiological Stage of fracture union : Callus

Answer. b

194. All of the following are associated as Pre Malignant lesions for Secondary Osteosarcoma Except :

a. Paget's Disease

b. Osteochondromatosis

c. Fibrous Dysplasia

d. Chondroblastoma

Solution. (d)
Explanation:
paget disease
Previous radiation treatment
endochondromatosis
Fibrous dysplasia
Osteochondromas
Osteogenesis imperfecta

Answer. d

195. Which is the most commonly involved muscle involved in Poliomyelitis :

a. Deltoid
b. Opponens Pollicis

c. Quadriceps Femoris

d. Tibialis Anterior

Solution. (c)
Explanation :
Most common muscle showing complete paralysis in Polio—Tibialis Anterior

Most common muscle of upper limb involved in Polio—Deltoid

Most common hand muscle to be involved in Polio- Opponens Pollicis

Most common muscle involved in PoliomyelitisPoliomyelitis—Quadriceps Femoris (partiallyparalyzed)

Answer. c

196. Identify the fracture in illustration :

a. Bumper's Fracture

b. Segond's Fracture

c. Hoffas' Fracture

d. Pellegrini Steida Fracture

Solution. (c)
Explanation :
Hoffas' Fracture - Coronal Plane Fracture of one or both femoral condyles

Answer. c

197. Identify the pathology shown in the image :

a. Gout
b. Pseudogout

c. Rheumatoid Arthritis

d. Osteoarthritis

Solution. (b)
Explanation - Intra Articular Calcification is a classical feature of Pseudogout

Answer. b

198. Tennis Elbow mainly involves :

a. Extensor Digitorum Brevis

b. Extensor Carpi Radialis Longus

c. Extensor Carpi Radialis Brevis

d. Extensor Digitorum Indicis

Solution. (c)
Explanation - Tennis Elbow mainly involves the Common Extensor Origin amongst which ECRB is most commonly involved

Answer. c

199. Identify the classification used in the fracture shown in the Xray :

a. Schatzker's Classification

b. Neer' Classification

c. Allman's Classification

d. Gartland's Classification

Solution. (b)
Explanation
Neer's Classification

Answer. b

200. All of the following are true about Sprengel’s Deformity except

a. Associated with Congenital Scoliosis

b. Associated with Diastemetomyelia


c. High incidence with Klippel Fiel Syndrome

d. Associated with Dextrocardia

Solution. (d)
Explanation
Sprengel deformity
It is congenital elevation of the scapula, is a complex deformity of the shoulder and is the most common congenital
shoulder abnormality.

Sprengel deformity is usually noticed at birth and has both cosmetic and
functional implications. The elevated scapula is visually noticeable and there is an
associated restriction in the motion of the scapula and glenohumeral joint.
Associations of Sprengel’s Deformity — An omovertebral bar (fibrous, cartilaginous and/or osseous connection
between the scapula and cervical spine) is often present. It is also commonly associated with hypoplasia or
atrophy of regional muscles, and these associated features can cause further misshaping of the shoulder and
limitation of shoulder movement. Patients with Sprengel deformity often have one or more of the following
abnormalities and conditions:
• Klippel-Feil syndrome
• Spina bifida & Diastemetomyelia
• Kyphoscoliosis
• Torticollis
• underdevelopment of clavicle or humerus

Answer. d

201. Which heart disease is most commonly associated with the following abnormality?

a. ASD

b. VSD

c. PDA

d. TGA

Solution. (b) VSD


Ref: Read the text below
Sol:
• This is an infant with hairy pinna. Hairy pinna is a classical morphological finding seen in infants of diabetic mothers.
• Among infants of diabetic mothers, VSD is most common congenital heart disease.

Answer. b

202. The correct sequence of congenital heart disease according to frequency of occurrence (most common to less
common) is?

a. VSD>ASD>TOF>PDA

b. VSD>PDA>Coarctation of aorta>ASD

c. VSD>ASD>Coarctation of aorta>d-TGA

d. VSD>PDA>ASD>d-TGA

Solution. (c) VSD>ASD>Coarctation of aorta>d-TGA


Ref: Nelson 20th Ed/2183
Sol:
Major congenital heart diseases frequency wise distribution
Answer. c

203. Which among the following is not a risk factor for future occurrence of epilepsy in a child with febrile seizures?

a. Family history of seizures

b. Focal complex febrile seizures

c. Recurrent febrile seizures

d. Onset at age<1 year

Solution. (d) Onset at age<1 year


Ref: Nelson Table 593-7
Sol:

Answer. d

204. In pediatric resuscitation, the preferred bone for obtaining intraosseous access is?

a. Anterior proximal tibia

b. Anterior distal tibia

c. Lower distal femur

d. Lower distal radius

Solution. (a) Anterior proximal tibia


Ref: Nelson 20th Ed, PALS-AHA)
Sol:
• IO cannulation is recommended for patients for whom IV access proves difficult or unattainable,even in older
children, irrespective of age.
• Intraosseous (IO) needles (for intramedullary venous plexus access) are special rigid, large-bore needles that
resemble those used for bone marrow aspiration.
• If venous access is not available within 1 min in a child with cardiopulmonary arrest, an IO needle should be
placed in the anterior proximal tibia (with care taken to avoid traversing the epiphyseal plate). The needle should
penetrate the anterior layer of compact bone, and its tip advanced into the spongy interior of the bone
• Any medications, blood products, and fluids may be administered through the IO route, including all medications
required for emergency resuscitation.
• Complications are uncommon, but may include osteomyelitis with prolonged infusions and tibial fracture.

Answer. a

205. What is the recurrence rate of Kawasaki disease in children?

a. Does not recur

b. 1-3%

c. 5-10%

d. 10-15%

Solution. (b) 1-3%


Ref: Read the text below
Sol:
Recurrence rate in KD is 1-3%.
Other high-yield points on KD:
• IVIG-resistant KD occurs in approximately 15% of patients and is defined by persistent or recrudescent fever 36 hr
after completion of the initial IVIG infusion. Therapy is to give 2nd dose of IVIG. If fever still fails to occur, give
Corticosteroids.
• Infliximab, Etanercept are reserve drugs in severe KD
• Ulinastatin is new drug for KD

Answer. b

206. Incorrect regarding management of hyaline membrane disease is?

a. CPAP is recommended in extreme preterm newborns at risk for HMD even if asymptomatic

b. Permissive hypercapnia is beneficial strategy for ventilation in HMD

c. High frequency oscillometric ventilation (HFOV) decreases risk of BPD and barotrauma

d. Caffeine citrate is beneficial in severe HMD

Solution. (d) Caffeine citrate is beneficial in severe HMD


Ref: Read the text below
Sol:
Management of hyaline membrane disease: Latest concepts (Nelson 20 th Ed, CPDT)
• CPAP reduces collapse of surfactant-deficient alveoliand improves both FRC and ventilation– perfusion matching. Early
useof CPAP for stabilization of at-risk preterm infants beginning as early asin the delivery room reduces ventilatory needs
• If an infant withRDS undergoing CPAP cannot keep oxygen saturation >90% whilebreathing 40- 70% oxygen, assisted
ventilation and surfactant areindicated.
• Infants with respiratory failure or persistent apnea require assistedmechanical ventilation. Reasonable measures of
respiratory failure are:(1) arterial blood pH <7.20, (2) arterial blood Pco2 of 60 mm Hg orhigher, and (3) oxygen
saturation <90% at oxygen concentrations of40-70% and CPAP of 5-10 cm H2O. Infants with persistent apnea alsoneed
mechanical ventilation.
• Intermittent positive pressure ventilation(IPPV) delivered by time-cycled, pressure-limited, continuous flow ventilatorsis
a common method of conventional ventilation for newborns. This is commonly done in SIMV (Synchronized intermittent
mechanical ventilation) mode.
• Permissive hypercapnia is a strategy for the management of patientsreceiving ventilatory support in which priority is
given to the preventionor limitation of lung injury from the ventilator by tolerating relativelyhigh levels of Paco2 rather
than maintenance of normal bloodgas values.
• The currently recommended range of oxygen saturationtargets is 91-95%.
• High-frequency ventilation (HFV) achieves desired alveolar ventilationby using smaller tidal volumes and higher
rates (300-1,200breaths/min or 5-20 Hz). HFV may improve elimination of carbondioxide and improve oxygenation in
patients who show no responseto conventional ventilators and those who have severe RDS, interstitialemphysema,
recurrent pneumothoraces, or meconium aspirationpneumonia. High-frequency oscillatory ventilation (HFOV) and
highfrequencyjet ventilation are the most frequently used methods ofHFV. HFOV reduces BPD but may raise the risk for
intracranial hemorrhage.
• Caffeine citrate has no role in the management of HMD but is an effective drug in the management of apnea of
prematurity.

Answer. d

207. Most rare cause of neonatal sepsis in India is?


a. E. coli

b. Neisseria meningitides

c. Staphylococcus aureus

d. Streptococcus agalactiae

Solution. (b) Neisseria meningitides

Ref:Read the text below

Sol:

Causes of neonatal sepsis

·      World (Compiled from Nelson, CPDT): Group B streptococcus (Streptococcus agalactiae) > E. coli >Klebsiella

·      India (NNF, AIIMS Protocol): Klebsiella> Staphylococcus aureus > Pseudomonas > E. coli > Others
(including Viridans streptococci, Staph albus, Enterococci, Acinetobacter and group B streptococcus)

·      Remember that Neisseria meningitides does not cause meningitis in neonates. This is a common cause of meningitis
in children beyond 2 years of age in India.

·      Also remember: Candida parapsilosis is MC fungal cause of neonatal sepsis in world as well as India

Answer. b

208. Chhotu, 50 day old child, with weight of 3.1 kg and fever since 2 days is brought to OPD. On examination, he
has respiratory rate of 62/min but no chest retractions or any other positive findings. What is the likely diagnosis?

a. No pneumonia

b. Pneumonia

c. Severe pneumonia

d. Very severe pneumonia

Solution. (c) Severe pneumonia


Ref: Read the text below
Sol:

Answer. c

209. Raju, a 5 year old kid, is having 5-6 spots on trunk as shown below and also has seizures, whose EEG has
also been shown. What shall be the drug of choice for Raju?
a. ACTH

b. Vigabatrin

c. Clonazepam

d. Valproate

Solution. (b) Vigabatrin


Ref: Read the text below
Sol:

Answer. b

210. A 5 year old child developed periorbital puffiness following an acute febrile illness. His spot urine protein
sample showed protein: creatinine ratio of 3.5:1. There was no hypertension or hematuria. The child was started
on oral prednisolone @ 2mg/kg/day as single morning dose. Despite 2 weeks of therapy, the child continues to have
periorbital edema and persistently elevated protein: creatinine ratio. What next needs to be done?

a. Continue prednisolone in same dose for 2 weeks more

b. Increase dose of prednisolone to double

c. Start Levamisole as steroid sparing agent

d. Consider renal biopsy

Solution. (a) Continue prednisolone in same dose for 2 weeks more

Ref:Read the text below

Sol:

Childhood nephrotic syndrome

·      Nephrotic syndrome in a childis said to be there when there is massive proteinuria (Urine protein excretion >40
mg/m2/hr or spot urine protein creatinine ratio is >2:1) with other features like hypoalbuminemia, edema,
hyperlipidemia and hypercoagulability of blood

·      A child aged 2 to 6 years and coming with only massive proteinuria and no hematuria/hypertension is treated on the
lines of minimal change disease. These children need no renal biopsy routinely and are started on oral prednisolone @ 2
mg/kg/day given for 6 weeks daily, followed by 1.5 mg/kg alternate day for 6 weeks

·      If child does not respond to daily steroids for 4 weeks (8 weeks in adults), he is called steroid-resistant and
renal biopsy is indicated

·      Since the child in ques has received steroids at full dose only for 2 weeks, we shall give for 2 more weeks before
labelling steroid resistance and doing biopsy.

·      Levamisole and cyclophosphamide are indicated in frequently relapsing and steroid dependent varieties

Answer. a

211. Which among the following parameters is not affected by severe malnutrition?

a. Delayed type hypersensitivity response

b. Total T cell count in blood

c. Total B cell count in blood

d. Complement levels in blood

Solution. -(b) Total T cell count in blood

Ref:Read the text below

Sol:

Features of malnutrition

Answer. b

212. Which among the following statement is false regarding the child shown in picture?

a. Caused due to autosomal recessive condition

b. Low serum zinc levels are seen

c. Low alkaline phosphatase levels are seen

d. Therapy with 0.3 mg/kg/day of zinc gluconate is recommended

Solution. (d) Therapy with 0.3 mg/kg/day of zinc gluconate is recommended

Ref:Read the text below

Sol:
This child has features of perioral rash with desquamating perianal rash. Typically seen in children with
acrodermatitisenteropathica.

Review of acrodermatitisenteropathica (Nelson 20th Ed)

·      Acrodermatitisenteropathica is a rare autosomal recessive disordercaused by an inability to absorb sufficient zinc
from the diet. Thegenetic defect is in the intestinal zinc-specific transporter geneSLC39A4.

·      Initial signs and symptoms usually occur in the 1st fewmonths of life, often after weaning from breast milk to cow’s
milk.

·      The cutaneous eruption consists of vesiculobullous, eczematous, dry, scaly,or psoriasiform skin lesions
symmetrically distributed in the perioral,acral, and perineal areas and on the cheeks, knees, andelbows

·      The hair often has a peculiar, reddish tint, andalopecia of some degree is characteristic.

·      Ocular manifestations includephotophobia, conjunctivitis, blepharitis, and corneal dystrophy detectableby
slit-lamp examination.

·      Associated manifestations includechronic diarrhea, stomatitis, glossitis, paronychia, nail dystrophy,growth
retardation, irritability, delayed wound healing, intercurrentbacterial infections, and superinfection with Candida albicans.

·      Lymphocytefunction and free radical scavenging are impaired.

·      The diagnosis is established by the constellation of clinical findingsand detection of a low plasma zinc concentration.
A serum zinc levelless than 50 μg/dL is suggestive

·      Levels of alkaline phosphatase, a zinc-dependentenzyme, may also be decreased.

·      Histopathologic changes in the skinare nonspecific and include parakeratosis and pallor of the upper epidermis.

·      Oral therapy with zinc compounds is the treatment of choice.Replacement for individuals with inherited
acrodermatitisenteropathicais with 3 mg/kg/24 hr of elemental zinc found in zinc sulfate, gluconate, or acetate

·      Plasma zinc levels should be monitored every 3-6 mo, however,to individualize the dosage. Zinc therapy rapidly
abolishes the manifestationsof the disease.

Answer. d

213. A child gets afraid of strangers and starts crying due to anxiety when mother is not visible. What is the
approximate age of the child?

a. 7 months

b. 9 months

c. 15 months

d. 18 months

Solution. (d) 18 months


Ref: Read the text below
Sol:
Answer. d

214.

Type of neonatal seizures with best prognosis?

a. Subtle

b. Focal clonic

c. Myoclonic

d. Tonic

Solution. (b) Focal clonic

Ref:Read the text below

Sol:

·      Among neonatal seizures, focal clonic type has best prognosis and myoclonic seizures have worst prognosis.

·      According to cause, late-onset hypocalcemic seizures have best and seizures due to asphyxia have worst
prognosis.

Remember: MC type of neonatal seizure is Subtle seizure but generally carries poor prognosis.

Extra edge: GTCS type of seizures are NOT SEEN in the neonatal period

Answer. b

215. A 2-year-old child cannot do which among the following?

a. Draw a vertical line

b. Making a tower of 6 cubes

c. Making a bridge of 3 cubes

d. Feeding self with spoon


Solution. (c) Making a bridge of 3 cubes

Ref:Read the text below

Sol:

Answer. c

216. A male child with 3 beta-hydroxysteroid dehydrogenase deficiency cannot have which among the following?

a. Hypoglycaemia

b. Normal genitalia

c. Increased DHEA

d. Decreased androstenedione

Solution. (b) Normal genitalia

Ref:Read the text below

Sol:

Answer. b

217. What shall be the most common haematological abnormality seen in the child shown in picture?
a. Aplastic anemia

b. Thrombocytopenia only

c. JMML

d. PNH

Solution.

(b) Thrombocytopenia only

Ref: Read the text below

Sol:

This is a child with absent radial bone (radial forearm defect), most likely due to TAR syndrome
(Thrombocytopenia with absent radii syndrome).

Note the presence of shortened/hypoplastic forearm with thumb being present (Fanconi anemia has absent thumb as
well).

Points of difference of TAR syndrome V/S Fanconi anemia (from MCQ perspective)
· In Fanconi anemia, thrombocytopenia is rarely present at birth; it develops later on in childhood or even in adulthood.
· Radial defects are seen in only 30% of patients with Fanconi anemia; when present, such defects are usually associated
with absent thumb, unlike in TAR, where radial defects are the sine qua non and the thumb is usually present, even
though it could be abnormal

Answer. b

218. Which among the following is most dangerous type of dehydration?

a. Hyponatremic

b. Isonatremic

c. Hypernatremic

d. None of the above

Solution. (c) Hypernatremic

Ref:Read the text below

Sol:

·      Hypernatremic dehydration(Serum Na >145 mEq/L) is the most dangerous form of dehydration because of
complications of hypernatremia and of therapy.

·      Hypernatremia

can cause serious neurologic damage, including central nervous system hemorrhages and thrombosis.

This damage appears to be secondary to the movement of water from the brain cells into the hypertonic extracellular
fluid, causing brain cell shrinkage and tearing blood vessels within the brain.

In this variety, ECF remains normal but ICF decreases (MCQ one-liner)

Children with hypernatremic dehydration are often lethargic, andthey may be irritable when touched. Hypernatremia may
cause fever,hypertonicity, and hyperreflexia. More severe neurologic symptoms may develop if cerebral bleeding or
thrombosis occurs.

Why slow correction recommended in this?

Idiogenic osmoles are generated within the brain during the development of hypernatremia. These idiogenic osmoles
increase the osmolality within the cells of the brain,providing protection against brain cell shrinkage caused by movement
of water out of the cells and into the hypertonic extracellular fluid. They dissipate slowly during the correction of
hypernatremia. With overly rapid lowering of the extracellular osmolality during the correction of hypernatremia, an
osmotic gradient may be created that causes water

movement from the extracellular space into the cells of the brain,producing cerebral edema. Symptoms of the resultant
cerebral edema can range from seizures to brain herniation and death.To minimize the risk of cerebral edema during the
correction

of hypernatremic dehydration, the serum sodium concentration should not decrease by >12 mEq/L every 24 hr. The
deficits in severe hypernatremic dehydration may need to be corrected over 2-4 days
Answer. c

219. Which among the following investigation(s) need to be done on follow-up in a 6 month old child treated
successfully for 1st episode of UTI?

a. USG-KUB

b. USG-KUB + DMSA

c. USG-KUB + DMSA + MCU

d. DMSA + MCU

Solution.

(c) USG-KUB + DMSA + MCU

Ref: Read the text below

Sol:

Answer. c

220. A preterm neonate has visible jaundice on day 24 of life. Which among the following cannot be a diagnosis?

a. Rh-incompatibility

b. Physiological jaundice

c. Breast milk jaundice

d. Extrahepatic biliary atresia

Solution. (b) Physiological jaundice

Ref:Read the text below

Sol:

Answer. b

221. What drug is used for prophylaxis against Pneumocystis jirovecii in patients on chemotherapy?
a. Cotrimoxazole

b. Amoxicillin

c. Dexamethasone

d. Cephalosporin

Solution. (a) Cotrimoxazole

Ref:Read the text below

Sol:

·      Pneumocystis jiroveciicauses severe pneumonia in neutropenic and AIDS patients.

·      Pneumocystis jiroveci pneumonia (PJP), formerly known as Pneumocystis cariniipneumonia (PCP), is the most
common opportunistic infection in persons with HIV infection

·      Drug of choice for prophylaxis and treatment of Pneumocystis jirovecii infection in both immuno-compromised
as well as immuno-competent is cotrimoxazole.

Answer. a

222. Gum hyperplasia, hirsutism and pseudolymphomais caused by ingestion of which drug

a. Phenobarbitone

b. Phenytoin

c. Carbamazepine

d. Diazepam

Solution. (b) Phenytoin

Ref:Read the text below

Sol:

§  Adverse effects of phenytoinare: Gum hypertrophy, hirsutism, hypersensitivity, megaloblastic anemia, osteomalacia,
and rarely a lymphoma like syndrome.

§  Used during pregnancy, phenytoin may cause: Fetal hydantoin syndrome comprising of hypoplastic phalanges, cleft
palate and microcephaly.

§  At high plasma levels, phenytoin may lead to: Cerebellar and ventricular manifestations – ataxia, vertigo, diplopia,
nystagmus, edema and gastric irritation.

Answer. b

223. Mechanism of action of Oseltamivir (Tamiflu) as an antiviral agent is:

a. Inhibition of M2 receptor

b. Neuraminidase inhibition

c. Inhibition of RNA dependent DNA polymerase

d. Apoptosis of infected cells

Solution. (b) Neuraminidase inhibition

Ref:Read the text below

Sol:

·      Oseltamivir acts by inhibiting influenza virus neuraminidase enzyme which is needed for release of progeny
virions from the infected cells.

·      Oseltamivir has a broad spectrum activity covering influenza-A, influenza-B and bird flu H5N1.
Answer. b

224. Etanercept acts by one of the following mechanisms:

a. By blocking tumor necrosis factor

b. By blocking bradykinin synthesis

c. By inhibiting cyclo-oxygenase-2

d. By blocking lipoxygenase

Solution. (a) By blocking tumor necrosis factor

Ref:Read the text below

Sol:

·      Etanercept (trade name Enbrel) is a drug that treats autoimmune diseases by interfering with the tumor necrosis
factor (TNF, a part of the immune system) by acting as a TNF inhibitor.

·      Etanercept is a fusion protein produced through expression of recombinant DNA.

·      That is, it is a product of a DNA "construct" engineered to link the human gene for soluble TNF receptor 2 to the
gene for the Fc component of human immunoglobulin G1 (IgG1).

Answer. a

225. Most effective bactericidal drug against mycobacterium leprae is:

a. Erythromycin

b. Ofloxacin

c. Cotrimoxazole

d. Amoxicillin

Solution. (b) Ofloxacin

Ref:Read the text below

Sol:

Anti-leprotic drugs:
Sulfones- Dapsone
Phenazine derivatives- Clofazimine
Anti-tubercular drugs- Rifampicin and Ethionamide

·      In recent studies, three newer antimicrobial agents—ofloxacin (OFLO; a fluoroquinolone), clarithromycin
(CLARI; a macrolide), and minocycline (MINO; a tetracycline derivative) demonstrated very promising
bactericidal activities against M. leprae
Others- Ofloxacin, Minocycline and Clarithromycin
Rifampicin is the only bactericidal first line drug against M. leprae

Answer. b

226. All of the following antihypertensives decrease plasma rennin activity, except

a. Clonidine

b. Methyldopa

c. Atenolol

d. Chlorthiazide

Solution. (d) Chlorthiazide

Ref:Read the text below

Sol:

§  Clonidine and methyldopa are central sympatholytics. Atenolol also decreases beta adrenergic pathway.

§  So, the above drugs decrease rennin release.

§  Diuretics like chlorthiazide increase rennin release.

Answer. d

227. Lente insulin is composed of:

a. 30 % amorphous + 70 % crystalline insulin

b. 30 % crystalline + 70 % amorphous insulin

c. Same as NPH insulin

d. Only 70 % amorphous insulin

Solution. (a) 30 % amorphous + 70 % crystalline insulin

Ref:Read the text below

Sol:

·      Lent insulinis composed of 30 % amorphous (Semi-lente and short acting) + 70 % crystalline (Ultra-lente and long
acting) insulin.

·      Lente insulin is intermediate acting.

Answer. a

228. Which is the first indication of distribution of a drug in tissues?

a. Large volume of distribution

b. Small volume of distribution

c. Renal excretion

d. Salivary excretion

Solution. (a) Large volume of distribution

Ref:Read the text below

Sol:

Indication of distribution of a drug in tissues:

·      Tissue sequestration


Large volume of distribution

·      Low renal excretion and excretion in other body fluids


Answer. a

229.

Which of the following disease modifying anti-rheumatoid drugs acts by causing extracellular accumulation of adenosine?

a. Leflunomide

b. Azathioprine

c. Hydroxychloroquine

d. Methotrexate

Solution. (d) Methotrexate

Ref:Read the text below

Sol:

Mode of action of Methotrexate:

·      In RA methotrexate inhibits enzymes involved in purine metabolism, leading to accumulation of adenosine.

·      In cancer, it competitively inhibits dihydrofolate reductase (DHFR) an enzyme that participates in the tetra-
hydrofolate synthesis.

Answer. d

230. Which of these drugs is an antidote for fibrinolytic therapy?

a. Epsilon amino-caproic acid

b. Protamine

c. Heparin

d. Streptokinase

Solution. (a) Epsilon amino-caproic acid

Ref:Read the text below

Sol:

·      Epsilon amino-caproic acid (EACA) combineswith lysine binding sites of plasminogen and plasmin and prevents
later binding with fibrin.

·      It is specific antidote for fibrinolytic therapy.

·      Tranexamic acid combineswith lysine binding sites of plasminogen and prevents its binding with fibrin.

·      Tranexamic acid is 7 times more potent.

Answer. a

231. A patient is administered 200 mg of a drug. 75 mg of the drug is excreted in 90 minutes. If the drug follows
first order kinetics, how much drug will be left after 6 hours?

a. 6.25 mg

b. 12.5 mg

c. 25 mg

d. 50 mg

Solution. (d) 50 mg

Ref:Read the text below

Sol:

Answer. d

232. Protease inhibitorsact on which of these processes in the human immune-virus?


a. Viral assembly

b. Pro-viral RNA synthesis

c. Viral entry into the cell

d. DNA synthesis from RNA

Solution. (a) Viral assembly


Ref:Read the text below
Sol:

Answer. a

233. A 64-year-old man presents with difficulty urinating and “dribbling” at the end of urination.Which of the
following drugs would be most appropriate for treating this man’s conditions?

a. Finasteride

b. Leuprolide

c. Mifepristone

d. Pergolide

Solution. (a) Finasteride

Ref:Read the text below

Sol:

§  Finasteride, an inhibitor of 5-alpha-reductase,prevents the conversion of testosterone to dihydrotestosterone


(DHT).

§  Because dihydrotestosterone is essential for the normal growth and development of the prostate gland, finasteride is
an effective treatment for benign prostatic hyperplasia, which is a DHT-dependent process.

Answer. a

234. A 33-year-old man receiving chemotherapy for testicular carcinoma develops signs of renal tubular damage.
Which of the following drugs is most likely responsible for this nephrotoxicity?

a. Bleomycin

b. Cisplatin

c. Cyclophosphamide
d. Vinblastine

Solution. (b) Cisplatin

Ref:Read the text below

Sol:

§  Cisplatin is an antineoplastic drugused in the treatment of carcinoma of the testes (along with bleomycin and
vinblastine) ovaries, bladder, and lung (especially small cell).

§  Along with the typical side effects of nausea, vomiting, and bone marrow suppression, cisplatin is notable for its
dose-limiting nephrotoxicity and ototoxicity.

Answer. b

235. All of the following are inhibitors of glucagon secretion except

a. Insulin

b. Hyperglycemia

c. Somatostatin

d. Amino acids.

Solution. (d) Amino acids.

Ref:Read the text below

Sol:

§  Amino acidsstimulate glucagon secretion. The effects of glucose on glucagon secretion are reciprocal to those of
insulin secretion in that hyperglycemia suppresses the secretion of glucagon.

§  Insulin is a potent inhibitor of glucagonsecretion and acts within the pancreatic islet.

§  Catecholamines and glucocorticoids, such as cortisol, inhibit glucagon secretion.

Answer. d

236. Idiosyncratic side-effects of carbamazepine are all; except:

a. Steven-Johnson syndrome

b. Agranulocytosis

c. Rash

d. Blurred vision

Solution. (d) Blurred vision

Ref:Read the text below

Sol:

Idiosyncratic side-effects of carbamazepine:

·      Rashes and Photosensitivity

·      Water retention and hypo-natremia

·      Hepatitis

·      Lupus like syndrome

·      Rarely Agranulocytosis and Aplastic anemia and leucopenia

✔   Blurred vision is dose related side effect.

Answer. d

237. Which of the following drugs is a substrate for P-glycoprotein?

a. Azithromycin
b. Ketoconazole

c. Itraconazole

d. Rifampicin

Solution. (c) Itraconazole

Ref:Read the text below

Sol:

P-gp is an ATP-dependent drug efflux pump for xenobiotic compoundswith broad substrate specificity. It is
responsible for decreased drug accumulation in multidrug-resistant cells and often mediates the development of
resistance to anticancer drugs.
Important point:
Drugs metabolized by cytochrome P450 (CYP) enzymes tend to also be substrates for P-gp.

Answer. c

238. A 70-year-old hypertensive patient with stage 5 chronic kidney disease was diagnosed recently with type-2
diabetes mellitus. He does not want to take injectable insulin. Which of the following oral hypoglycemic agents will
be preferred in this patient, which won’t require any renal dose modification?

a. Linagliptin

b. Vildagliptin

c. Repaglinide

d. Glimepiride

Solution. (a) Linagliptin

Ref:Read the text below

Sol:

·      Linagliptin-Excreted by liver and can be given renal failure


·      Repaglinide- Excreted by kidney and can be given in hepatic failure

·      Glimepiride is contraindicated in renal and live failure

Answer. a

239. A drug X was given continuous intravenous infusion at 1.6 mg/ min. The clearance of the drug is 640 ml/min.
With a half-life of 1.8 hours, what would be the steady state plasma concentration of drug?

a. 2.88 mg/ml

b. 0.004 mg/ml

c. 0.002 mg/ml

d. 3.55 mg/ml

Solution. (c) 0.002 mg/ml

Ref:Read the text below

Sol:

Answer. c

240. An extensive infection with trichophyton rubrum would best be treated with

a. Tolnaftate

b. Griseofulvin

c. Nystatin

d. Miconazole

Solution. (b) Griseofulvin

Ref:Read the text below

Sol:

§  Trichophyton rubrumcauses superficial fungal infection (dermatophytosis)

§  Mild to moderate superficial mycoses is treated with topical agents like miconazole, nystatin, tolnaftate etc.
moderate to severe superficial mycoses (dermatophytosis) requires an oral agent like griseofulvin (for best effect, in
addition of topical agents.) 

Answer. b

241. The motility of cells is due to( neet)

a. Motilin

b. Tubulin

c. Laminin

d. Tactilin

Solution. b

CYTOSKELETON

A) Microtubules( 25nm): tubulin

     shape, motility, mitotic  spindle**.

e.g. cilia, flagella, axon and dendrites

B) Microfilaments(7-9.5 nm): e.g, microvilli, F-actin , lamellopodia.

C) Intermediate filaments( 10-12 nm):

e.g. laminin , keratins, neurofilaments, vimentin ,desmin, peripherins.

Answer. b
242. Which of the following methods is not used for measurement of body fluid volumes;

a. Antipyrine for total body water

b. Inulin for ECF

c. Evans blue for plasma volume

d. I 125 albumin for blood volume

Solution. d

TBW- Deuterium oxide(D2O), Tritium oxide.

              Antipyrine and aminopyrine

ECF: inulin  *, sucrose, mannitol, sodium              thiosulphate ,

Plasma volume- Evan’s blue( T-1824), serum albumin labelled with radioactive iodine.(I125)

Blood cell volume  -Cr labelled RBC

ICF and IF cannot be measured by dilution method.

•       ICF = TBW- ECF

•       IF= ECF-Plasma volume

Answer. d

243. At the end of isometric relaxation phase:

a. Atrioventricular valves open

b. Atrioventricular valves close

c. Corresponds to peak of “C” wave in JVP

d. Corresponds to “T” Wave in ECG

Solution. a

•          Isometric(isovolumic)  relaxation phase comes to an end with opening of AV valves.

Opening of AV valves: end of Iso.vol.metric relaxation  and start of rapid filling phase.

•          Closure of AV valves: end of diastole and  start of isovolumetric contraction period.

•          Opening of SML valve: end of isovolmetric contraction ,Start of ejection phase

•          Closure of Semilunar Valves : End of systole,Beginning of isovolumetric relaxation

Answer. a

244. C waves in JVP is due to;

a. Atrial contraction

b. Atrial filling with tricuspid valves closed

c. Rapid right ventricular filling

d. Bulging of tricuspid valve into the right atrium

Solution. d

JVP:

a – atrial systole ( corresponds to the last rapid filling phase of ventricles)

c – bulging of the tricuspid valve in the RA( isovolumetric contraction period of ventricles).

v- filling of the atria ( protodiastole of ventricles)

x- atrial muscles get stretched & atria dilate ( ventricular ejection).

y – rapid ventricular filling


Answer. d

245. All the following factors normally increase the length of the ventricular cardiac muscle fibers except;

a. Increased venous tone

b. Increased total blood volume

c. Increased negative intrathoracic pressure

d. Lying to standing change in posture

Solution. d

Initial length of cardiac muscle : starling law

C0 = HR X SV

Initial length, increased :

§  Venous tone ( sympathetic activity, decreases in venous capacitance)

§  Respiratory pump

§  Cardiac pump

§  Skeletal muscle pump

§  Increase in blood volume

§  Body posture ( supine)

Answer. d

246. Gastric secretion is increased by all except:

a. Histamine

b. A.ch

c. Gastrin

d. HCl

Solution. d

Gastric acid secretion :

Increased by;

Histamine( H2)***

A.ch( M3), vagal stimulation .

Gastrin

Decreased by:

•       Somatostanin

•       Secretin

•       CCK

•       GIP, VIP

•       PGE2

•       LOW pH. ( Hcl)

Answer. d

247. Which of the following statements regarding secretin is least correct;

a. Inhibits gastric acid secretion

b. Increase bicarbonate rich pancreatic secretion


c. Causes contraction of pyloric spinchter

d. Increases gastric acid secretion

Solution. d

Secretin :

•       Stimulus: acid in the stomach*

•       S cells: duodenum & jejeunum

•       MOA : cAMP ( group 2 A).

•       Actions: stimulation of watery alkaline  pancreatic secretion ( HCO3- ,poor in enzymes).

Actions :

•       Potentiates the action of CCK on pancreas.

•       Increase hepatic secretion of bile

•       Increases the tone of pyloric spinchter

•       Inhibit gastric emptying & secretion.

Answer. d

248. Which of the following sensory receptors is found in epidermis for determination of texture

a. Merkel disc

b. Meissner’s corpuscles

c. Paccinian corpuscles

d. Ruffini ending

Solution. a

Receptors

1. Superficial (epidermis)-merkel’s disc(detects 2 point discrimination)**, meissner’s  corpuscles (texture of surface).


2. Deep (dermis)- ruffini end organ(sustained pressure), paccinian corpuscles(vibrtaion).

Rapidly adapting/movement/rate/phasic: e.g

Meissner’s corpuscles& paccinian corpuscles.

Slowly/tonic: e.g.  Merkel’s disc & ruffini end organ.

Answer. a

249. Vestibulo ocular reflex is concerned with;

a. Paleo cerebellum

b. Neocerebellum

c. Flocculunodular lobe

d. Occipital lobe

Solution. c

Divisions of cerebellum:

1)  Vestibulocerebellum /archicerebellum/flocculonodular lobe;

Oldest,

Postural equilibrium & balance.

Coordination of eye movements with head.

Spino cerebellum /paleocerebellum/ includes  vermis & medial portions of cerebellum;

 smoothens & coordinates movements. 


3)Neocerebellum/ cerebrocerebellum /includes lateral portions of cerebellum

Newest part,

Planning & programming of movements.

Answer. c

250. In presence of vasopressin the greatest fraction of filtered water is reabsorbed in which part of nephron:

a. Proximal tubule

b. Distal tubule

c. Loop of Henle

d. Collecting duct

Solution. a

•       Water reabsorption:

80-85% 0f water reabsorption shows constant pattern( obligatory), PCT( 65%)  % thin DL of LOH(15%). Aquaporin-1(
ADH independent).

15-20% - facultative ( ADH) , late DT % CD , aquaporin -2.

Maximum H20 reabsorption- PCT.

Answer. a

251. Skeletal muscle contraction end when-

a. Ions move out of the cytoplasm

b. Ach absorbed from NMJ

c. Closure and in drawing of receptors

d. Decreased calcium outside reticulum.

Solution. a

RELAXATION OF MUSCLE IS BY :

Reuptake of Ca++ by sarcoplasmic reticulum,

During relaxation;

1)Cytoplasmic concentration of Ca++ decreases

2)Sarcoplasmic reticulum concentration of Ca++ increases.

Answer. a

252. A 36-year-old male experiences symptoms of tachycardia, palpitations, and an irregular heart beat, particularly at
night. His plasma catecholamine levels are found to be increased, which may result from which of the following?

a. An increase in blood pressure

b. An increase in blood volume

c. An increase in plasma cortisol

d. An increase in blood glucose

Solution. (c)

An increase in plasma cortisol Ref– Read the text below Sol: • Phenylethanolamine-Nmethyltransferase (PNMT), the
enzyme that catalyzes the formation of epinephrine from norepinephrine, is found in appreciable quantities only in the
brain and the adrenal medulla. • Adrenal medullary PNMT is induced by glucocorticoiods and glucocorticoids are
necessary for the normal development of the adrenal medulla. • Circumstances that increase sympathetic nerve input to
the adrenal medulla increase catecholamine secretion. • Major stressors include decreased intravascular volume or
pressure, fear or rage, a change in posture from supine to standing, and hypoglycemia.

Answer. c
253. Golgi tendon apparatus conveys message to CNS depends upon;

a. Tension in muscle

b. Length of muscle

c. Rapidity of contraction

d. Blood supply

Solution. a

•       Golgi tendon( collection of knobby nerve endings),detects muscle tension. has about 3-25 muscle fibres, they
are present in series with the muscle fibers.

•       Innervations:  A α afferent/type 1b, which innervate .

•       It brings the inverse stretch reflex, also known as the autogenic inhibition or protective reflex. Thus
it leads to muscle relaxation.

Answer. a

254. Total alveolar ventilation volume( L/m)

a. 1.5

b. 3.5

c. 4.2

d. 5.0

Solution. c

•          Alveolar ventilation is ,

 tidal volume -  dead space volume  X  RR  ; 500- 150 X 12 = 4.2 L/min

•          Minute volume/ Pulmonary ventilation/Respiratory minute volume=

     TV X RR =500 X 12 =6 L/min

Answer. c

255. Greatest stimulator for ADH secretion is-

a. Hyperosmolarity

b. Hyponatremia

c. Hypotension

d. Hypovolemia

Solution. a

Synthesized : supraoptic nucleus * along with neurophysin1*

Most important stimulus : plasma hyperosmolarity .

Action :   water reabsorption in CD & DCT ( aquaporin 2*).

Answer. a

256. Deep Brain stimulation is useful in :


a. Parkinson’s

b. Depression

c. Essential Tremors

d. All of above

Solution. (d) All of above

Ref:Read the text below

Sol:

DBS Indications:

a.       Parkinson’s

b.      Depression

c.       Essential Tremors

d.      OCD

e.       Dystonia

f.       Tourette’s

Answer. d

257. Most common psychiatric diseaseseen in cancer patients?

a. Depression

b. Adjustment disorder

c. Anxiety disorder

d. Acute stress disorder

Solution. (b) Adjustment disorder

Ref:Read the text below

Sol:

·         Adjustment disorder is most common in cancer patients.


Answer. b

258. All of the following are classified as gateway substances except?

a. Tobacco

b. Alcohol

c. Marijuana

d. Heroin

Solution. D Heroin

National anti-drug programs such as Drug Abuse Resistance Education (D.A.R.E.) specifically outline the consequences of
three potential gateway drugs: marijuana, alcohol and tobacco.

Answer. d

259. Which of the following culture bound syndromes seen in india presents with acute anxiety and panic like
symptoms?

a. Koro syndrome

b. Dhat syndrome

c. Amok syndrome

d. Possession syndrome

Solution.

A Koro syndrome

Koro syndrome presents with acute anxiety about decreasing in size of genitalia.

Amok syndrome presents with violent, aggressive and homicidal behaviour

Dhat syndrome presents with getting weaker due to loss of semen.

Possession syndrome presents with multiple presentations including dramatizing behaviour.

Answer. a

260. In which of the following stages of Sigmund freud’s psychosexual stages of development, does the male
child develops a fear of getting castration?

a. Oral

b. Anal

c. Phallic

d. Genital

Solution. C Phallic

Phallic stage in males : oedipal complex (attraction towards mother ) and castration anxiety (fear of castration)

Phallic stage in females :electra complex and penis envy

Answer. c

261. During a psychiatric interview a patient was asked, “What is the thing to do, if you find an envelope in the
street that is sealed, stamped and addressed?”. Which part of the mental status examination here is being
tested?

a. Attention

b. Abstract Ability

c. Insight level

d. Judgement
Solution. D Judgement

In testing for judgement, we give the patient a hypothetical test situation as mentioned in the question above.

Answer. d

262. During the psychiatric interview, a person was asked “what is the similarity between a table and a chair,
and he replied that both have four legs.” What is the thinking abnormality most likely in this patient?

a. Impaired judgement

b. Good Abstract ability

c. Concretisation of thinking

d. Normal healthy thinking

Solution. C Concretization of thinking

In testing for abstract ability, we do similarity test and when the person gives functional similarity like both are furniture,
he has good abstract ability. And when the person gives structural similarity like both have four legs, that means patient
has poor abstract ability, which is also known as concretisation of thinking.

Answer. c

263. A 13-year-old boy is brought to the emergency room by his parentsafter he set fire to their home. He has been seen
in the emergency room on multiple occasions for a variety of symptoms, including suicidality,
homicidality,uncontrollable tantrums, and pica. Of those symptoms, which ismost commonly seen by psychiatrists in
the emergency room in childrenunder the age of 12?

a. Aggression

b. Suicidality

c. Homicidality

d. Uncontrollable tantrums

Solution. (b) Suicidality

Ref:Read the text below

Sol:

·         Suicidal behavior is the most commonreason for a psychiatrist to see an adolescent in an emergency roomsetting.

·         Important in the evaluation of these children is an assessment ofthe stability and supportiveness of the
home environment, and the caregiver’s competence in taking care of the adolescent.

·         These factors will figurein to a clinician’s decision as to whether a potentially suicidal adolescentmust be admitted
to an inpatient unit or may be released home to beclosely monitored.

Answer. b

264. La belle indifference is seen in :

a. Conversion Disorder

b. Depression

c. Schizophrenia

d. All of the above

Solution. A. Conversion Disorder

La Belle Indifférence. La belle indifférenceis a patient’s inappropriately cavalier attitude toward serious symptoms; that
is, the patient seems to be unconcerned about what appears to be a major impairment. That bland indifference is also seen
in some seriously ill medical patients who develop a stoic attitude. The presence or absence of la belle indifférenceis not
pathognomonic of conversion disorder, but it is often associated with the condition.

Answer. a
265. Most common eating disorder?

a. Anorexia Nervosa

b. Bulimia Nervosa

c. Binge eating disorder

d. Obesity

Solution. C Binge eating disorder

Answer. c

266. Most common hormone deficiency seen in children after intracranial radiation therapy

a. Prolactin

b. Gonadotropin

c. ACTH

d. Growth hormone

Solution. (d) Growth hormone.

Ref:Read the text below

Sol:

Cranial irradiation – GH deficiency is the most common endocrinopathy seen in childhood cancer survivors
following cranial irradiation.

Answer. d

267. "Bright" lesion on T2WI MRI imagewith imperceptible walls is :

a. Pnematocele

b. Simple renal cyst

c. Chronic liver haematoma

d. Heterotopia

Solution. (b) Simple renal cyst.

Ref:Read the text below

Sol:

·   Fluid or water is bright on T2WI and out of the choices only simple renal cyst is a fluid containing lesion and will
appear hyperintense on T2WI and will have imperceptible walls.

Answer. b

268. Identify the structure marked.


a. Vallecula

b. Piriform sinus

c. Epiglottis

d. Glossoepiglottic fold

Solution. (b) Piriform sinus.

Ref:Read the text below

Sol:

Note the corresponding anatomicaldiagram labelled

Answer. b

269. Lady with trauma for evaluation underwent a neck xray  &  CXR, and later discovered to be 10 week pregnant ,
what will you explain to her?

a. Amniocentesis is required
b. USG guided chorionic villi biopsy is required

c. Pregnancy needs to be terminated

d. Reassurance

Solution. (d) Reassurance

Ref:Read the text below

Sol:

·   Reassurance as  the radiation exposure in these xrays is minimal and is not a matter of concern.

·   However, please remember,CT scan has much more radiation exposure than these and could be a matter of concern.

·   Radiation dose after which teratogenesis is seen is 5Rad.

Answer. d

270. Nonvisualization of gallbladder in hepatic scintigraphyis suggestive of:

a. Chronic cholocystitis

b. Carcinoma gallladder c

c. Acute chloecystitis due to gallstones

d. Gallstones obstructing due to gallstones

Solution. (c) Acute chloecystitis due to gallstones

Ref:Read the text below

Sol:

·   HIDA cholescintigraphy in acute cholecystitis will demonstrate non-visualisation of the gallbladder as the cystic duct
is obstructed due to GB wall edema in this patient. However, note IOC for acute cholecystitis is USG now.

·   Cholescintigraphy (HIDA scan) —Cholescintigraphy using 99mTc-hepatic iminodiacetic acid (generically referred to
as a HIDA scan) is indicated if the diagnosis remains uncertain following ultrasonography. Technetium labeled hepatic
iminodiacetic acid (HIDA) is injected intravenously and is then taken up selectively by hepatocytes and excreted into bile.
If the cystic duct is patent, the tracer will enter the gallbladder, leading to its visualization without the need for
concentration.

·   The HIDA scan is also useful for demonstrating patency of the common bile duct and ampulla. Normally, visualization
of contrast within the common bile duct, gallbladder, and small bowel occurs within 30 to 60 minutes. The test is positive
if the gallbladder does not visualize. This occurs because of cystic duct obstruction, usually from edema associated
with acute cholecystitis or an obstructing stone.

Answer. c

271. Below is radioactive testing scan for thyroid pathology. Which of the following is the most probable diagnosis?

a. Papillary ca thyroid

b. Hypersecreting adenoma

c. Graves disease

d. Lateral aberrant thyroid


Solution. (b) Hypersecreting adenoma.

Ref:Read the text below

Sol:

Tc-99m [pertechnetate] thyroid scan

Dose and route of administration 3-5 mCi IV

·   Timing of imaging – 20 minutes after Tc-99m pertechnetate administration

·   Camera: gamma camera

·   Collimator: pinhole

·   Window: 20% energy window centered at 140 KeV

Increased uptake noted in the right lobe of thyroid gland along with suppressed in left lobe of thyroid

Answer. b

272. Linear accelator for

a. Electrons and x-rays

b. Gamma and protons

c. Electrons and protons

d. Neutrons and xray

Solution. (a) Electrons and x-rays

Ref:Read the text below

Sol:
·   Linear accelerator (LINAC) isthe device used for external beam radiation treatments for patients with cancer.

·   It delivers high-energy x-rays or electrons to the region of the patient's tumor.

Answer. a

273. Identify structures marked in the image:

a. CT scan showing A= ACL and B= PCL

b. MRI showing A=ACL and B= PCL

c. CT scan showing A= PCL and B= ACL

d. MRI scan showing A= PCL and B= ACL

Solution. (b) MRI showing A=ACL and B= PCL.

Ref:Read the text below

Sol:

MRI showing A=ACL and B= PCL

This is a sagittal MRI image of the Knee, with “Cortex” of the bone appearing dark and subcutaneous fat appearing white.
A is pointing towards the Anterior cruciate ligament, and B is pointing at posterior cruciate ligament.

Answer. b

274. IOC for Intraocular foreign body?

a. CT scan

b. MRI

c. USG

d. X-Ray

Solution. (a) CT scan.

Ref:Read the text below

Sol:

·   Axial and coronal computed tomography (CT) of the eye without contrast, utilizing 1 to 2 mm cuts through the
orbits, is the preferred modality for further assessment of suspected open globe injuries.

·   CT is superior to ultrasoundin assessing the location and size of intraocular foreign bodies (IOFBs) and requires
no direct contact with the eyelids or globe.

·   CT is faster than magnetic resonance imaging (MRI), has less motion artifact, andwill not cause movement of
metallic foreign bodies.

Answer. a
275. Maximum radiation exposure is seenin which of the following:

a.

b.

c.

d.

Solution. (b) CT abdomen.

Ref:Read the text below

Sol:

Note the images show MRI brain which obviously has no radiation exposure and CXR which is very little
radiation. CT abdomen has more radiation than Barium enema (d) choice.

Answer. b

276. Normal Level of PSA in male is?

< 4 ng/ml>a.

b. 4-10 ng/ml

c. >10 ng/ml

d. PSA is not produced by normal males

Solution. (a) < 4 ng/ml


Ref– Read the text below
Sol:

Answer. a

277. A patient suffered from 3rd degree burn of right upper limb, 2nd degree burns of right lower limb and 1st
degree burn of whole of the back. Total percentage of burn will be?

a. 27%

b. 36%

c. 45%

d. 54%

Solution. (a) 27%


Ref– Sabiston 18th e chapter 22, Practical plastic surgery by Zol B.Kryger, Mark Sisco page 155

Sol: 

·   Superficial Burns (First Degree) are not included in the assessment of the TBSA of a burn victim.

·   Burn size is generally assessed by the so-called rule of nines. In adults, each upper extremity and the head and neck are
9% of TBSA (total body surface area), the lower extremities and the anterior and posterior aspects of the trunk are
18% each, and the perineum and genitalia are assumed to be 1% of TBSA.

·   In this case there is 9% of 3rd degree burn, 18% of 2nd degree burn and 18%of 1st degree burn. So TBSA
involved in burns will be 9+18=27% burns.

Answer. a

278. Stain used to diagnose premalignant lesion of lip is?

a. Crystal violet

b. H & E

c. Toluidine blue

d. Giemsa

Solution. (c) Toluidine blue

Ref– Read the text below

Sol: 

·   Mitotic indexis a clinically important parameter in cancer pathology.

·   Selective histochemical stains like toluidine blue, Giemsa and crystal violet have been used in tissues including the
developing brain, neutral tissue and skin.

·   Toluidine Blue detect efficientlyand rapidly mitotic figures in sections of formalin fixed paraffin – embedded human
tissues especially in oral cavity.

Answer. c

279.

The pre-malignant condition with the highest probability of progression to malignancy is :

a. Dysplasia

b. Hyperplasia

c. Leucoplakia

d. Erythroplakia

Solution. (d) Erythroplakia

Ref.:Read the text below

Sol :

·   “ Malignant potential is 17 times higher in erythroplakia “.

·   “ Dysplasia does not necessarily progress to cancer”

·   “ Leucoplakia and erythroplakia are precancerous condition.”

Answer. d

280. Structure damaged most commonly during Surgery on Ranula is?

a. Lingual Artery

b. Lingual Nerve

c. Submandibular duct

d. Sublingual Duct
Solution. (c) Submandibular duct        

Ref– Read the text below

Sol: 

·   “ The treatment of ranula constitutes a problem, owing to the technical difficulty of complete excision without
damage to adjacent structures such as the submandibular duct.”

Answer. c

281. Which one of the following gastrointestinal disorders predisposes to urolithiasis ?

a. Peutz-jegher’s syndrome

b. Short bowel syndrome

c. Familial polyposis coli

d. Ulcerative colitis

Solution. (b) Short bowel syndrome

Ref.:Read the text below

Sol :

§Short bowel syndrome may developafter extensive resection of small intestine like trauma, etc. necrotizing
enterocolitis and congenital atresia are most common pediatric causes.

§Calcium oxalate calculi occur in patients who have extensive ileal resection with intact colon resulting from
excessive absorption of oxalate from colon.

Answer. b

282. What is the most important aspect of management of  burn injury in  the first 24 hours ?

a. Fluid resuscitation

b. Dressing

c. Escharotomy

d. Antibiotics

Solution. (a) Fluid resuscitation

Ref.:Schwartz’s Principle of Surgery, (8th Ed.), Pg. 199, 212

Sol :

§In first 24 hours, most important aspect of management of burn is fluid resuscitation because injured
microvasculature may manifest increase permeability and rate of fluid loss is maximum in first 24 hours.

§Most patient can be resuscitated with crystalloid specifically Ringer lactate.

Answer. a

283. A man who had undergone total gastrectomy remains well for five years. Then he develops anaemia. What is
the most likely cause of this anaemia :

a. Protein deficiency

b. Zinc deficiency

c. Folic acid deficiency

d. Vitamin B12 deficiency

Solution. (d) Vitamin B12 deficiency

Ref.:Bailey and Love’s Short Practice of Surgery, (24the Ed.), Pg 105-07

Sol :

§Total gastrectomy :
oFat absorption defect – 70%.

oImpairment in protein absorption – 60%.

oVarious gastric substitutes, mostly using Roux-en-Y principle, most popular of these reservoir is Hunt-Lawrence pouch.

oNeeded supplementary vitamin B12 administration on a continuing monthly basis to prevent megaloblastic anemia.

Answer. d

284. A patient with blunt abdominal trauma presented to casualty with BP 90/60 mm Hg & pulse rate 124. The
investigation to be done in this patient is?

a. CT abdomen

b. MRI abdomen

c. DPL

d. FAST

Solution. (d) FAST

Ref– Read the text below

Sol: 

·   As the above mentioned patient is in shock with history of blunt abdomen trauma, the first investigation to be
done should be FAST.

·   “Ultrasound has an evolving role in the assessment of acutely traumatized patients.The main current roles of
ultrasound include the assessment of intraperitoneal fluid and haemopericardium (focused assessment with
sonography for trauma, FAST), the evaluation of pneumothoraces in supine patients and in guiding
intervention.”

·   Fast ultrasound is a limited examination directed examination to look for intraperitoneal fluid or pericardial injury as a
marker of underlying injury.

·   This avoids the invasiveness of diagnostic peritoneal lavage. In the presence of free intraperitoneal fluid and an
unstable patient, the ultrasound allows the trauma surgeon to explore the abdomen as a cause of blood loss.

·   In the presence of fluid and a haemodynamically stable individual, further assessment by way of CT can be
performed.

·   Occasionally, a second ultrasound scan may show free fluid in the presence of an initially negative FAST scan.

Answer. d

285. Complications of chronic pancreatitis include the following except :

a. Portal hypertension

b. Obstructive jaundice

c. Duodenal obstruction

d. Renal artery aneurysm

Solution. (d) Renal artery aneurysm

Ref.:Read the text below

Sol : 

§Complications of chronic pancreatitis :

oPseudocysts and pancreatic ascites

oExtrahepatic obstructive jaundice due to benign stricture of common bile duct

oDuodenal stenosis

oPortal and splenic vein thrombosis leading to segmental portal hypertension and gastric varices

oPeptic ulcer.

Answer. d
286. In which one of the following conditions is gas under diaphragm not seen ?

a. Perforated duodenal ulcer

b. Typhoid perforation

c. After laparotomy

d. Spontaneous rupture of oesophagus

Solution. (d) Spontaneous rupture of oesophagus

Ref.:Read the text below

Sol :  

·   Pneumoperitoneum :

oErect – free gas under diaphragm or liver. Can detect 10 ml of gas.

oSupine – gas outlines both sides of bowel wall, which then appears as a white line.

oPerforation – (a) Peptic ulcer (30% do not have free air visible), (b) Inflammation – diverticultis, appendicitis, toxic
megacolon, necrotizing enterocolitis, (c) Infraction, (d) Malignant neoplasm, (e) Obstruction, (f) Pneumatosis coli (cysts
may rupture).

oIatrogenic (surgery : peritoneal dialysis) – may take 3 weeks to reabsorb (faster in obese and children)

oPneumomediastinum

Answer. d

287. Best method to treat a large port –wine hemangioma is?

a. Radiotherapy

b. Tatooing

c. Excision with skin grafting

d. Pulsed dye Laser

Solution. (d) Pulsed dye Laser 

Pulsed dye Laser 

Ref– Read the text below

Sol: 

Pulsed dye Laser 

·   “Selective Photothermolysis or Pulsed dye laser is the treatment of choice for Portwine hemangioma.”

·   Port – wine stains are crimson blotches in which there is marked capillary dilation compared to  a capillary angioma,
which is a red nodule or plaque containing proliferating endothelial cells.

·   The latter tend to flatten and disappear at puberty.

·   Larger ones may cause problems from bleeding and/ or erosion.

·   Cavernous haemangiomata are larger and compressible containing larger vascular spaces.

Answer. d

288. Tube-enterostomy is indicated when enteral feeding is required for more than:

a. 1 week

b. 2 weeks.

c. 4 weeks.

d. 6 weeks.

Solution. (d) 6 weeks.

Ref:Read the text below


Sol:

·   When internal feeding is likely to be required for less than 6 weeks, a narrow-bore feeding tube can be inserted via
the nasal route.

·   Tube enterostomy is indicated for longer term feeding and should be considered when there is a likely need for
prolonged postoperative nutritional support in a patient undergoing laparotomy.

Answer. d

289. What is the most common complication of Meckel’s diverticulum among adults?

a. Bleeding

b. Perforation

c. Intestinal obstruction

d. Ulceration

Solution. (c) Intestinal obstruction

Ref:Read the text below

Sol:

·   Intestinal obstruction due to a Meckel’sdiverticulum may result from a volvulus, band obstruction, or
intussusception.

·   Among children, bleeding and inflammation are seen more frequently. Meckel’s diverticulum is a remnant of the
vitellointestinal duct.

Answer. c

290. One side of kidney is normal, other side kidney is contracted kidney with scar, what is the most probable
diagnosis?

a. Chronic pyelonephritis

b. Polycystic kidney

c. Renal artery stenosis

d. Tuberculosis of Kidney

Solution.  (a) Chronic pyelonephritis                              

Ref– Read the text below

Sol: 

Differential Diagnosis of a Granular, contracted Kidney with scar is:

1.Chronic Glomerulonephritis 

2.Chronic Pyelonephritis           

3.Benign Nephrosclerosis (Hypertension)

Answer. a

291. Regarding burst abdomen, which of the following condition carries high risk?

a. Non-absorbable suture

b. Interrupted suture

c. Median incision

d. Transverse incision

Solution. (c) Median incision

Ref.:Read the text below

Sol :
Answer. c

292. All electrolyte abnormalities are seen in immediate postoperative period, except

a. Negative Nitrogen balance

b. Hypokalemia

c. Glucose intolerance

d. Hypernatremia

Solution. (d) Hypernatremia

Ref.: Read the text below

Sol :

§Acute severe hyponatermia occasionallydevelops in patients undergoing elective surgery.

§In these patients the hyponatremia results from excessive administration of sodium free fluid (e.g. D5%) coupled
with post surgical stimulation of ADH release. Premenopausal women are at greater risk.

§Other options are seen in immediate postoperative period.

Answer. d

293. Which of the following most common cause of hypotension in fracture ribs (T10-T12) ?

a. Abdominal solid visceral organ injury

b. Injury to aorta

c. Inter costal artery damage

d. Pulmonary contusion

Solution.  (a) Abdominal solid visceral organ injury

Ref.:Read the text below

Sol :

Answer. a

294. The following have strong casual association with H. pylori infection  except
a. Chronic gastritis

b. Peptic ulcer disease

c. Gastric carcinoma

d. Gastric adenoma

Solution. (d) Gastric adenoma

Ref: Read the text below.

Sol :

§Associations of H. pylori :

oMajority of peptic ulcer disease.

oGastric mucosal-associated lymphoid tissue (MALT) lymphoma

oGastric adenocarcinoma.

Answer. d

295. The treatment of cholecysto-enteric fistula following ulceration of gallstones through the gall bladder into the
duodenum

a. Excision of track and closure of fistula

b. Divert the bile of another anastomosis

c. Cholecystesctomy & primary repair of the bowel

d. Radical excision of all related structures and reconstruction

Solution. (c) Cholecystectomy & primary repair of the bowel

Ref: Read the text below

Sol :

§Fistulization into an adjacent organadherent to the acutely inflamed gallbladder wall may result from inflammation
and adhesion formation.

§Fistulas into the duodenum are most common.

§When gallstones are passed directly through the fistula into the small bowel, if they are greater than 2.5 cm, they can
obstruct the ileocecal valve. This causes gallstone ileus.

§Clinically “silent” biliary-enteric fistulas occurring as a complication of acute cholecystitis have been found in up to
5% of patients undergoing cholecystectomy.

Answer. c

296. Which of the following liver lesions carry a significant risk of spontaneous rupture?

a. Hemagioma

b. Hepatic cyst

c. Adenoma

d. Bile duct hamartoma

Solution. C

Hepatic adenoma is a benign tumor of liver... associated with OCP intake...ten times more common in females...
spontaneous rupture and bleeding are complications...it can turn into malignancy

Answer. c

297. A 62 yr old woman presents to the ED with a 2 days history of severe left lower quadrant abdominal pain
and is found to be febrile to 39 degree Celsius. On Physical examination her abdomen is rigid. Her WBC count
is 21000. On CT scan she is found to have diverticula and gross intra-ab dominal free air and free fluid. She is
taken to the operating  Room(OR) for an emergent exploratory laparotomy and She is found to have Feculent
Material Intra-abdominally . What is her Hinchey Stage?
a. Stage I

b. Stage II

c. Stage III

d. Stage IV

Solution. D

Hinchey classification is for perforated colonic diverticula

I - pericolic abscess

II - localised pelvic abscess

III - generalised purulent peritonitis

IV - generalised feculent peritonitis

Answer. d

298. The Triangle of pain is bordered by all of the following EXCEPT

a. Ilipubic tract

b. Ductus Deferens

c. Gonadal Vessels

d. Reflected Peritoneum

Solution. B

Triangle of pain is an important region during laparoscopic hernia repair... important nerves are present in this region
which can get trapped under tackers during mesh fixation which results in significant discomfort in post op period

Answer. b

299. Supplementation of which of the following micronutrients improves wound healing  in  patients without
micronutrient deficiency?

a. Vitamin C

b. Vitamin A

c. Selenium

d. Zinc

Solution. B

Although all the options are important in wound healing but except vit A none of them add any extra advantage in
supraphysiological dose..

Answer. b

300. During an emergency appendectomy, a surgical resident sustains an injury from a contaminated hollow
-bore needle with spontaneous bleeding. Which one of the following blood -borne organisms is most likely to
be transmitted, assuming that patient was infected with all of them ?

a. HIV

b. HBV

c. HCV

d. Plasmodium spp Malaria

Solution. C

Hepatitis C has the maximum risk of transmission in needle stick injury...it is around 30%

Answer. c

Test Answer
1. (c) 2. (c) 3. (d) 4. (b) 5. (a) 6. (b) 7. (d) 8. (d) 9. (a) 10. (b)
11. (c) 12. (c) 13. (a) 14. (d) 15. (d) 16. (c) 17. (b) 18. (a) 19. (b) 20. (b)
21. (d) 22. (c) 23. (d) 24. (d) 25. (b) 26. (a) 27. (d) 28. (b) 29. (a) 30. (d)
31. (c) 32. (c) 33. (d) 34. (c) 35. (a) 36. (a) 37. (c) 38. (c) 39. (c) 40. (b)
41. (c) 42. (c) 43. (a) 44. (d) 45. (a) 46. (d) 47. (c) 48. (c) 49. (d) 50. (a)
51. (c) 52. (a) 53. (b) 54. (b) 55. (a) 56. (d) 57. (b) 58. (c) 59. (d) 60. (b)
61. (a) 62. (b) 63. (d) 64. (a) 65. (b) 66. (c) 67. (b) 68. (b) 69. (c) 70. (c)
71. (b) 72. (c) 73. (a) 74. (a) 75. (b) 76. (a) 77. (b) 78. (d) 79. (a) 80. (c)
81. (d) 82. (a) 83. (d) 84. (b) 85. (d) 86. (b) 87. (a) 88. (c) 89. (a) 90. (d)
91. (a) 92. (d) 93. (c) 94. (a) 95. (a) 96. (c) 97. (a) 98. (b) 99. (b) 100. (b)
101. (b) 102. (a) 103. (d) 104. (b) 105. (a) 106. (b) 107. (c) 108. (c) 109. (c) 110. (c)
111. (d) 112. (c) 113. (d) 114. (c) 115. (d) 116. (b) 117. (b) 118. (d) 119. (c) 120. (d)
121. (c) 122. (a) 123. (b) 124. (b) 125. (b) 126. (c) 127. (c) 128. (c) 129. (d) 130. (b)
131. (a) 132. (d) 133. (b) 134. (b) 135. (a) 136. (d) 137. (b) 138. (d) 139. (d) 140. (c)
141. (c) 142. (c) 143. (a) 144. (a) 145. (b) 146. (c) 147. (c) 148. (b) 149. (d) 150. (a)
151. (c) 152. (b) 153. (d) 154. (a) 155. (a) 156. (d) 157. (a) 158. (c) 159. (b) 160. (b)
161. (d) 162. (c) 163. (a) 164. (b) 165. (b) 166. (b) 167. (a) 168. (d) 169. (c) 170. (b)
171. (d) 172. (d) 173. (b) 174. (b) 175. (b) 176. (c) 177. (d) 178. (d) 179. (b) 180. (b)
181. (d) 182. (d) 183. (d) 184. (d) 185. (d) 186. (a) 187. (c) 188. (a) 189. (c) 190. (c)
191. (d) 192. (b) 193. (b) 194. (d) 195. (c) 196. (c) 197. (b) 198. (c) 199. (b) 200. (d)
201. (b) 202. (c) 203. (d) 204. (a) 205. (b) 206. (d) 207. (b) 208. (c) 209. (b) 210. (a)
211. (b) 212. (d) 213. (d) 214. (b) 215. (c) 216. (b) 217. (b) 218. (c) 219. (c) 220. (b)
221. (a) 222. (b) 223. (b) 224. (a) 225. (b) 226. (d) 227. (a) 228. (a) 229. (d) 230. (a)
231. (d) 232. (a) 233. (a) 234. (b) 235. (d) 236. (d) 237. (c) 238. (a) 239. (c) 240. (b)
241. (b) 242. (d) 243. (a) 244. (d) 245. (d) 246. (d) 247. (d) 248. (a) 249. (c) 250. (a)
251. (a) 252. (c) 253. (a) 254. (c) 255. (a) 256. (d) 257. (b) 258. (d) 259. (a) 260. (c)
261. (d) 262. (c) 263. (b) 264. (a) 265. (c) 266. (d) 267. (b) 268. (b) 269. (d) 270. (c)
271. (b) 272. (a) 273. (b) 274. (a) 275. (b) 276. (a) 277. (a) 278. (c) 279. (d) 280. (c)
281. (b) 282. (a) 283. (d) 284. (d) 285. (d) 286. (d) 287. (d) 288. (d) 289. (c) 290. (a)
291. (c) 292. (d) 293. (a) 294. (d) 295. (c) 296. (c) 297. (d) 298. (b) 299. (b) 300. (c)

Das könnte Ihnen auch gefallen